Sei sulla pagina 1di 21

TEST I - Foundation of Professional Nursing Practice a. “My ankle looks less swollen now”.

b. “My ankle feels warm”.


1. The nurse In-charge in labor and delivery unit administered a dose c. “My ankle appears redder now”.
of terbutaline to a client without checking the client’s pulse. The d. “I need something stronger for pain relief”
standard that would be used to determine if the nurse was negligent
is: 10.The physician prescribes a loop diuretic for a client. When
a. The physician’s orders. administering this drug, the nurse anticipates that the client may
b. The action of a clinical nurse specialist who is recognized expert in develop which electrolyte imbalance?
the field. a. Hypernatremia b. Hyperkalemia c. Hypokalemia d. Hypervolemia
c. The statement in the drug literature about administration of
terbutaline. 11.She finds out that some managers have benevolent-authoritative
d. The actions of a reasonably prudent nurse with similar education style of management. Which of the following behaviors will she
and experience. exhibit most likely?
a. Have condescending trust and confidence in their subordinates.
2. Nurse Trish is caring for a female client with a history of GI b. Gives economic and ego awards.
bleeding, sickle cell disease, and a platelet count of 22,000/μl. The c. Communicates downward to staffs.
female client is dehydrated and receiving dextrose 5% in half-normal d. Allows decision making among subordinates.
saline solution at 150 ml/hr. The client complains of severe bone
pain and is scheduled to receive a dose of morphine sulfate. In 12. Nurse Amy is aware that the following is true about functional
administering the medication, Nurse Trish should avoid which route? nursing
a. I.V b. I.M c. Oral d. S.C a. Provides continuous, coordinated and comprehensive nursing
services.
3. Dr. Garcia writes the following order for the client who has been b. One-to-one nurse patient ratio.
recently admitted “Digoxin .125 mg P.O. once daily.” To prevent a c. Emphasize the use of group collaboration.
dosage error, how should the nurse document this order onto the d. Concentrates on tasks and activities.
medication administration record?
a. “Digoxin .1250 mg P.O. once daily” 13.Which type of medication order might read "Vitamin K 10 mg I.M.
b. “Digoxin 0.1250 mg P.O. once daily” daily × 3 days?"
c. “Digoxin 0.125 mg P.O. once daily” a. Single order b. Standard written order
d. “Digoxin .125 mg P.O. once daily” c. Standing order d. Stat order

4. A newly admitted female client was diagnosed with deep vein 14.A female client with a fecal impaction frequently exhibits which
thrombosis. Which nursing diagnosis should receive the highest clinical manifestation?
priority? a. Increased appetite b. Loss of urge to defecate
a. Ineffective peripheral tissue perfusion related to venous c. Hard, brown, formed stools d. Liquid or semi-liquid stools
congestion.
b. Risk for injury related to edema. 15.Nurse Linda prepares to perform an otoscopic examination on a
c. Excess fluid volume related to peripheral vascular disease. female client. For proper visualization, the nurse should position the
d. Impaired gas exchange related to increased blood flow. client's ear by:
a. Pulling the lobule down and back
5. Nurse Betty is assigned to the following clients. The client that the b. Pulling the helix up and forward
nurse would see first after endorsement? c. Pulling the helix up and back
a. A 34 year-old post operative appendectomy client of five hours d. Pulling the lobule down and forward
who is complaining of pain.
b. A 44 year-old myocardial infarction (MI) client who is complaining 16.Which instruction should nurse Tom give to a male client who is
of nausea. having external radiation therapy:
c. A 26 year-old client admitted for dehydration whose intravenous a. Protect the irritated skin from sunlight.
(IV) has infiltrated. b. Eat 3 to 4 hours before treatment.
d. A 63 year-old post operative’s abdominal hysterectomy client of c. Wash the skin over regularly.
three days whose incisional dressing is saturated with d. Apply lotion or oil to the radiated area when it is red or sore.
serosanguinous fluid.
17.In assisting a female client for immediate surgery, the nurse In-
6. Nurse Gail places a client in a four-point restraint following orders charge is aware that she should:
from the physician. The client care plan should include: a. Encourage the client to void following preoperative medication.
a. Assess temperature frequently. b. Explore the client’s fears and anxieties about the surgery.
b. Provide diversional activities. c. Assist the client in removing dentures and nail polish.
c. Check circulation every 15-30 minutes. d. Encourage the client to drink water prior to surgery.
d. Socialize with other patients once a shift.
18. A male client is admitted and diagnosed with acute pancreatitis
7. A male client who has severe burns is receiving H2 receptor after a holiday celebration of excessive food and alcohol. Which
antagonist therapy. The nurse In-charge knows the purpose of this assessment finding reflects this diagnosis?
therapy is to: a. Blood pressure above normal range.
a. Prevent stress ulcer b. Block prostaglandin synthesis b. Presence of crackles in both lung fields.
c. Facilitate protein synthesis. d. Enhance gas exchange c. Hyperactive bowel sounds
d. Sudden onset of continuous epigastric and back pain.
8. The doctor orders hourly urine output measurement for a
postoperative male client. The nurse Trish records the following 19.Which dietary guidelines are important for nurse Oliver to
amounts of output for 2 consecutive hours: 8 a.m.: 50 ml; 9 a.m.: 60 implement in caring for the client with burns?
ml. Based on these amounts, which action should the nurse take? a. Provide high-fiber, high-fat diet
a. Increase the I.V. fluid infusion rate b. Provide high-protein, high-carbohydrate diet.
b. Irrigate the indwelling urinary catheter c. Monitor intake to prevent weight gain.
c. Notify the physician d. Provide ice chips or water intake.
d. Continue to monitor and record hourly urine output
20.Nurse Hazel will administer a unit of whole blood, which priority
9. Tony, a basketball player twist his right ankle while playing on the information should the nurse have about the client?
court and seeks care for ankle pain and swelling. After the nurse a. Blood pressure and pulse rate. b. Height and weight.
applies ice to the ankle for 30 minutes, which statement by Tony c. Calcium and potassium levels d. Hgb and Hct levels.
suggests that ice application has been effective?
21. Nurse Michelle witnesses a female client sustain a fall and 32.Which of the following item is considered the single most
suspects that the leg may be broken. The nurse takes which priority important factor in assisting the health professional in arriving at a
action? diagnosis or determining the person’s needs?
a. Takes a set of vital signs. a. Diagnostic test results b. Biographical date c. History of present
b. Call the radiology department for X-ray. illness d. Physical examination
c. Reassure the client that everything will be alright. 33.In preventing the development of an external rotation deformity
d. Immobilize the leg before moving the client. of the hip in a client who must remain in bed for any period of time,
the most appropriate nursing action would be to use:
22.A male client is being transferred to the nursing unit for a. Trochanter roll extending from the crest of the ileum to the
admission after receiving a radium implant for bladder cancer. The midthigh.
nurse in-charge would take which priority action in the care of this b. Pillows under the lower legs.
client? c. Footboard
a. Place client on reverse isolation. d. Hip-abductor pillow
b. Admit the client into a private room.
c. Encourage the client to take frequent rest periods. 34.Which stage of pressure ulcer development does the ulcer extend
d. Encourage family and friends to visit. into the subcutaneous tissue?
a. Stage I b. Stage II c. Stage III d. Stage IV
23.A newly admitted female client was diagnosed with
agranulocytosis. The nurse formulates which priority nursing 35.When the method of wound healing is one in which wound edges
diagnosis? are not surgically approximated and integumentary continuity is
a. Constipation b. Diarrhea c. Risk for infection d. Deficient restored by granulations, the wound healing is termed
knowledge a. Second intention healing b. Primary intention healing
c. Third intention healing d. First intention healing
24.A male client is receiving total parenteral nutrition suddenly
demonstrates signs and symptoms of an air embolism. What is the 36.An 80-year-old male client is admitted to the hospital with a
priority action by the nurse? diagnosis of pneumonia. Nurse Oliver learns that the client lives
a. Notify the physician. alone and hasn’t been eating or drinking. When assessing him for
b. Place the client on the left side in the Trendelenburg position. dehydration, nurse Oliver would expect to find:
c. Place the client in high-Fowlers position. a. Hypothermia b. Hypertension
d. Stop the total parenteral nutrition. c. Distended neck veins d. Tachycardia

25.Nurse May attends an educational conference on leadership 37.The physician prescribes meperidine (Demerol), 75 mg I.M. every
styles. The nurse is sitting with a nurse employed at a large trauma 4 hours as needed, to control a client’s postoperative pain. The
center who states that the leadership style at the trauma center is package insert is “Meperidine, 100 mg/ml.” How many milliliters of
task-oriented and directive. The nurse determines that the meperidine should the client receive?
leadership style used at the trauma center is: a. 0.75 b. 0.6 c. 0.5 d. 0.25
a. Autocratic. b. Laissez-faire.
c. Democratic. d. Situational 38.A male client with diabetes mellitus is receiving insulin. Which
statement correctly describes an insulin unit?
26.The physician orders DS 500 cc with KCl 10 mEq/liter at 30 cc/hr. a. It’s a common measurement in the metric system.
The nurse in-charge is going to hang a 500 cc bag. KCl is supplied 20 b. It’s the basis for solids in the avoirdupois system.
mEq/10 cc. How many cc’s of KCl will be added to the IV solution? c. It’s the smallest measurement in the apothecary system.
a. .5 cc b. 5 cc c. 1.5 cc d. 2.5 cc d. It’s a measure of effect, not a standard measure of weight or
quantity.
27.A child of 10 years old is to receive 400 cc of IV fluid in an 8 hour
shift. The IV drip factor is 60. The IV rate that will deliver this amount 39.Nurse Oliver measures a client’s temperature at 102° F. What is
is: the equivalent Centigrade temperature?
a. 50 cc/ hour b. 55 cc/ hour c. 24 cc/ hour d. 66 cc/ hour a. 40.1 °C b. 38.9 °C c. 48 °C d. 38 °C

28.The nurse is aware that the most important nursing action when 40.The nurse is assessing a 48-year-old client who has come to the
a client returns from surgery is: physician’s office for his annual physical exam. One of the first
a. Assess the IV for type of fluid and rate of flow. physical signs of aging is:
b. Assess the client for presence of pain. a. Accepting limitations while developing assets.
c. Assess the Foley catheter for patency and urine output b. Increasing loss of muscle tone.
d. Assess the dressing for drainage. c. Failing eyesight, especially close vision.
d. Having more frequent aches and pains.
29.Which of the following vital sign assessments that may indicate
cardiogenic shock after myocardial infarction? 41.The physician inserts a chest tube into a female client to treat a
a. BP – 80/60, Pulse – 110 irregular b. BP – 90/50, Pulse – 50 regular pneumothorax. The tube is connected to water-seal drainage. The
c. BP – 130/80, Pulse – 100 regular d. BP – 180/100, Pulse – 90 nurse in-charge can prevent chest tube air leaks by:
irregular a. Checking and taping all connections.
b. Checking patency of the chest tube.
30.Which is the most appropriate nursing action in obtaining a blood c. Keeping the head of the bed slightly elevated.
pressure measurement? d. Keeping the chest drainage system below the level of the chest.
a. Take the proper equipment, place the client in a comfortable
position, and record the appropriate information in the client’s 42.Nurse Trish must verify the client’s identity before administering
chart. medication. She is aware that the safest way to verify identity is to:
b. Measure the client’s arm, if you are not sure of the size of cuff to a. Check the client’s identification band.
use. c. Have the client recline or sit comfortably in a chair with the b. Ask the client to state his name.
forearm at the level of the heart. c. State the client’s name out loud and wait a client to repeat it.
d. Document the measurement, which extremity was used, and the d. Check the room number and the client’s name on the bed.
position that the client was in during the measurement.
43.The physician orders dextrose 5 % in water, 1,000 ml to be
31.Asking the questions to determine if the person understands the infused over 8 hours. The I.V. tubing delivers 15 drops/ml. Nurse
health teaching provided by the nurse would be included during John should run the I.V. infusion at a rate of:
which step of the nursing process? a. 30 drops/minute b. 32 drops/minute
a. Assessment b. Evaluation c. Implementation d. Planning and c. 20 drops/minute d. 18 drops/minute
goals
44.If a central venous catheter becomes disconnected accidentally, 58.Nurse Amy has an order to obtain a urinalysis from a male client
what should the nurse in-charge do immediately? with an indwelling urinary catheter. The nurse avoids which of the
a. Clamp the catheter b. Call another nurse c. Call the physician d. following, which contaminate the specimen?
Apply a dry sterile dressing to the site. a. Wiping the port with an alcohol swab before inserting the syringe.
45.A female client was recently admitted. She has fever, weight loss, b. Aspirating a sample from the port on the drainage bag. c.
and watery diarrhea is being admitted to the facility. While assessing Clamping the tubing of the drainage bag. d. Obtaining the specimen
the client, Nurse Hazel inspects the client’s abdomen and notice that from the urinary drainage bag.
it is slightly concave. Additional assessment should proceed in which 59.Nurse Meredith is in the process of giving a client a bed bath. In
order: the middle of the procedure, the unit secretary calls the nurse on
a. Palpation, auscultation, and percussion. b. Percussion, palpation, the intercom to tell the nurse that there is an emergency phone call.
and auscultation. c. Palpation, percussion, and auscultation. d. The appropriate nursing action is to:
Auscultation, percussion, and palpation. a. Immediately walk out of the client’s room and answer the phone
Nursing Crib – Student Nurses’ Community 14 call. b. Cover the client, place the call light within reach, and answer
46. Nurse Betty is assessing tactile fremitus in a client with the phone call. c. Finish the bed bath before answering the phone
pneumonia. For this examination, nurse Betty should use the: call. d. Leave the client’s door open so the client can be monitored
a. Fingertips b. Finger pads c. Dorsal surface of the hand d. Ulnar and the nurse can answer the phone call.
surface of the hand 60. Nurse Janah is collecting a sputum specimen for culture and
47. Which type of evaluation occurs continuously throughout the sensitivity testing from a client who has a productive cough. Nurse
teaching and learning process? Janah plans to implement which intervention to obtain the
a. Summative b. Informative c. Formative d. Retrospective specimen?
48.A 45 year old client, has no family history of breast cancer or a. Ask the client to expectorate a small amount of sputum into the
other risk factors for this disease. Nurse John should instruct her to emesis basin.
have mammogram how often? Nursing Crib – Student Nurses’ Community 17
a. Twice per year b. Once per year c. Every 2 years d. Once, to b. Ask the client to obtain the specimen after breakfast. c. Use a
establish baseline sterile plastic container for obtaining the specimen. d. Provide
49.A male client has the following arterial blood gas values: pH 7.30; tissues for expectoration and obtaining the specimen.
Pao2 89 mmHg; Paco2 50 mmHg; and HCO3 26mEq/L. Based on 61. Nurse Ron is observing a male client using a walker. The nurse
these values, Nurse Patricia should expect which condition? determines that the client is using the walker correctly if the client:
a. Respiratory acidosis b. Respiratory alkalosis c. Metabolic acidosis a. Puts all the four points of the walker flat on the floor, puts weight
d. Metabolic alkalosis on the hand pieces, and then walks into it. b. Puts weight on the
50.Nurse Len refers a female client with terminal cancer to a local hand pieces, moves the walker forward, and then walks into it. c.
hospice. What is the goal of this referral? Puts weight on the hand pieces, slides the walker forward, and then
a. To help the client find appropriate treatment options. b. To walks into it. d. Walks into the walker, puts weight on the hand
provide support for the client and family in coping with terminal pieces, and then puts all four points of the walker flat on the floor.
illness. c. To ensure that the client gets counseling regarding health 62.Nurse Amy has documented an entry regarding client care in the
care costs. d. To teach the client and family about cancer and its client’s medical record. When checking the entry, the nurse realizes
treatment. that incorrect information was documented. How does the nurse
Nursing Crib – Student Nurses’ Community 15 correct this error?
51.When caring for a male client with a 3-cm stage I pressure ulcer a. Erases the error and writes in the correct information. b. Uses
on the coccyx, which of the following actions can the nurse institute correction fluid to cover up the incorrect information and writes in
independently? the correct information. c. Draws one line to cross out the incorrect
a. Massaging the area with an astringent every 2 hours. b. Applying information and then initials the change. d. Covers up the incorrect
an antibiotic cream to the area three times per day. c. Using normal information completely using a black pen and writes in the correct
saline solution to clean the ulcer and applying a protective dressing information
as necessary. d. Using a povidone-iodine wash on the ulceration 63.Nurse Ron is assisting with transferring a client from the
three times per day. operating room table to a stretcher. To provide safety to the client,
52.Nurse Oliver must apply an elastic bandage to a client’s ankle and the nurse should:
calf. He should apply the bandage beginning at the client’s: a. Moves the client rapidly from the table to the stretcher. b.
a. Knee b. Ankle c. Lower thigh d. Foot Uncovers the client completely before transferring to the stretcher.
53.A 10 year old child with type 1 diabetes develops diabetic c. Secures the client safety belts after transferring to the stretcher.
ketoacidosis and receives a continuous insulin infusion. Which d. Instructs the client to move self from the table to the stretcher.
condition represents the greatest risk to this child? 64.Nurse Myrna is providing instructions to a nursing assistant
a. Hypernatremia b. Hypokalemia c. Hyperphosphatemia d. assigned to give a bed bath to a client who is on contact precautions.
Hypercalcemia Nurse Myrna instructs the nursing assistant to use which of the
54.Nurse Len is administering sublingual nitrglycerin (Nitrostat) to following protective items when giving bed bath?
the newly admitted client. Immediately afterward, the client may a. Gown and goggles b. Gown and gloves c. Gloves and shoe
experience: protectors d. Gloves and goggles
a. Throbbing headache or dizziness b. Nervousness or paresthesia. c. Nursing Crib – Student Nurses’ Community 18
Drowsiness or blurred vision. d. Tinnitus or diplopia. 65. Nurse Oliver is caring for a client with impaired mobility that
55.Nurse Michelle hears the alarm sound on the telemetry monitor. occurred as a result of a stroke. The client has right sided arm and
The nurse quickly looks at the monitor and notes that a client is in a leg weakness. The nurse would suggest that the client use which of
ventricular tachycardia. The nurse rushes to the client’s room. Upon the following assistive devices that would provide the best stability
reaching the client’s bedside, the nurse would take which action for ambulating?
first? a. Crutches b. Single straight-legged cane c. Quad cane d. Walker
a. Prepare for cardioversion b. Prepare to defibrillate the client c. 66.A male client with a right pleural effusion noted on a chest X-ray
Call a code d. Check the client’s level of consciousness is being prepared for thoracentesis. The client experiences severe
Nursing Crib – Student Nurses’ Community 16 dizziness when sitting upright. To provide a safe environment, the
56.Nurse Hazel is preparing to ambulate a female client. The best nurse assists the client to which position for the procedure?
and the safest position for the nurse in assisting the client is to a. Prone with head turned toward the side supported by a pillow. b.
stand: Sims’ position with the head of the bed flat. c. Right side-lying with
a. On the unaffected side of the client. b. On the affected side of the the head of the bed elevated 45 degrees. d. Left side-lying with the
client. c. In front of the client. d. Behind the client. head of the bed elevated 45 degrees.
57.Nurse Janah is monitoring the ongoing care given to the potential 67.Nurse John develops methods for data gathering. Which of the
organ donor who has been diagnosed with brain death. The nurse following criteria of a good instrument refers to the ability of the
determines that the standard of care had been maintained if which instrument to yield the same results upon its repeated
of the following data is observed? administration?
a. Urine output: 45 ml/hr b. Capillary refill: 5 seconds c. Serum pH: a. Validity b. Specificity c. Sensitivity d. Reliability
7.32 d. Blood pressure: 90/48 mmHg
68.Harry knows that he has to protect the rights of human research 83.Which of the following theory addresses the four modes of
subjects. Which of the following actions of Harry ensures adaptation?
anonymity? a. Madeleine Leininger b. Sr. Callista Roy c. Florence Nightingale d.
a. Keep the identities of the subject secret b. Obtain informed Jean Watson
consent c. Provide equal treatment to all the subjects of the study. 84.Ms. Garcia is responsible to the number of personnel reporting to
d. Release findings only to the participants of the study her. This principle refers to:
69.Patient’s refusal to divulge information is a limitation because it is a. Span of control b. Unity of command c. Downward
beyond the control of Tifanny”. What type of research is appropriate communication d. Leader
for this study? 85.Ensuring that there is an informed consent on the part of the
a. Descriptive- correlational b. Experiment c. Quasi-experiment d. patient before a surgery is done, illustrates the bioethical principle
Historical of:
Nursing Crib – Student Nurses’ Community 19 a. Beneficence b. Autonomy c. Veracity d. Non-maleficence
70.Nurse Ronald is aware that the best tool for data gathering is? Nursing Crib – Student Nurses’ Community 22
a. Interview schedule b. Questionnaire c. Use of laboratory data d. 86.Nurse Reese is teaching a female client with peripheral vascular
Observation disease about foot care; Nurse Reese should include which
71.Monica is aware that there are times when only manipulation of instruction?
study variables is possible and the elements of control or a. Avoid wearing cotton socks. b. Avoid using a nail clipper to cut
randomization are not attendant. Which type of research is referred toenails. c. Avoid wearing canvas shoes. d. Avoid using cornstarch on
to this? feet.
a. Field study b. Quasi-experiment c. Solomon-Four group design d. 87.A client is admitted with multiple pressure ulcers. When
Post-test only design developing the client's diet plan, the nurse should include:
72.Cherry notes down ideas that were derived from the description a. Fresh orange slices b. Steamed broccoli c. Ice cream d. Ground
of an investigation written by the person who conducted it. Which beef patties
type of reference source refers to this? 88.The nurse prepares to administer a cleansing enema. What is the
a. Footnote b. Bibliography c. Primary source d. Endnotes most common client position used for this procedure?
73.When Nurse Trish is providing care to his patient, she must a. Lithotomy b. Supine c. Prone d. Sims’ left lateral
remember that her duty is bound not to do doing any action that will 89.Nurse Marian is preparing to administer a blood transfusion.
cause the patient harm. This is the meaning of the bioethical Which action should the nurse take first?
principle: a. Arrange for typing and cross matching of the client’s blood. b.
a. Non-maleficence b. Beneficence c. Justice d. Solidarity Compare the client’s identification wristband with the tag on the
74.When a nurse in-charge causes an injury to a female patient and unit of blood. c. Start an I.V. infusion of normal saline solution. d.
the injury caused becomes the proof of the negligent act, the Measure the client’s vital signs.
presence of the injury is said to exemplify the principle of: 90.A 65 years old male client requests his medication at 9 p.m.
a. Force majeure b. Respondeat superior c. Res ipsa loquitor d. instead of 10 p.m. so that he can go to sleep earlier. Which type of
Holdover doctrine nursing intervention is required?
Nursing Crib – Student Nurses’ Community 20 a. Independent b. Dependent c. Interdependent d. Intradependent
75.Nurse Myrna is aware that the Board of Nursing has quasi-judicial 91.A female client is to be discharged from an acute care facility
power. An example of this power is: after treatment for right leg thrombophlebitis. The Nurse Betty
a. The Board can issue rules and regulations that will govern the notes that the client's leg is pain-free, without redness or edema.
practice of nursing b. The Board can investigate violations of the The nurse's actions reflect which step of the nursing process?
nursing law and code of ethics c. The Board can visit a school Nursing Crib – Student Nurses’ Community 23
applying for a permit in collaboration with CHED d. The Board a. Assessment b. Diagnosis c. Implementation d. Evaluation
prepares the board examinations 92.Nursing care for a female client includes removing elastic
76.When the license of nurse Krina is revoked, it means that she: stockings once per day. The Nurse Betty is aware that the rationale
a. Is no longer allowed to practice the profession for the rest of her for this intervention?
life b. Will never have her/his license re-issued since it has been a. To increase blood flow to the heart b. To observe the lower
revoked c. May apply for re-issuance of his/her license based on extremities c. To allow the leg muscles to stretch and relax d. To
certain conditions stipulated in RA 9173 d. Will remain unable to permit veins in the legs to fill with blood.
practice professional nursing 93.Which nursing intervention takes highest priority when caring for
77.Ronald plans to conduct a research on the use of a new method a newly admitted client who's receiving a blood transfusion?
of pain assessment scale. Which of the following is the second step a. Instructing the client to report any itching, swelling, or dyspnea. b.
in the conceptualizing phase of the research process? Informing the client that the transfusion usually take 1 ½ to 2 hours.
a. Formulating the research hypothesis b. Review related literature c. Documenting blood administration in the client care record. d.
c. Formulating and delimiting the research problem d. Design the Assessing the client’s vital signs when the transfusion ends.
theoretical and conceptual framework 94.A male client complains of abdominal discomfort and nausea
78.The leader of the study knows that certain patients who are in a while receiving tube feedings. Which intervention is most
specialized research setting tend to respond psychologically to the appropriate for this problem?
conditions of the study. This referred to as : a. Give the feedings at room temperature. b. Decrease the rate of
a. Cause and effect b. Hawthorne effect c. Halo effect d. Horns effect feedings and the concentration of the formula. c. Place the client in
79.Mary finally decides to use judgment sampling on her research. semi-Fowler's position while feeding. d. Change the feeding
Which of the following actions of is correct? container every 12 hours.
a. Plans to include whoever is there during his study. b. Determines 95.Nurse Patricia is reconstituting a powdered medication in a vial.
the different nationality of patients frequently admitted and decides After adding the solution to the powder, she nurse should:
to get representations samples from each. c. Assigns numbers for a. Do nothing. b. Invert the vial and let it stand for 3 to 5 minutes. c.
each of the patients, place these in a fishbowl and draw 10 from it. Shake the vial vigorously. d. Roll the vial gently between the palms.
Nursing Crib – Student Nurses’ Community 21 96.Which intervention should the nurse Trish use when
d. Decides to get 20 samples from the admitted patients administering oxygen by face mask to a female client?
80.The nursing theorist who developed transcultural nursing theory a. Secure the elastic band tightly around the client's head. b. Assist
is: the client to the semi-Fowler position if possible. c. Apply the face
a. Florence Nightingale b. Madeleine Leininger c. Albert Moore d. Sr. mask from the client's chin up over the nose.
Callista Roy Nursing Crib – Student Nurses’ Community 24
81.Marion is aware that the sampling method that gives equal d. Loosen the connectors between the oxygen equipment and
chance to all units in the population to get picked is: humidifier.
a. Random b. Accidental c. Quota d. Judgment 97.The maximum transfusion time for a unit of packed red blood
82.John plans to use a Likert Scale to his study to determine the: cells (RBCs) is:
a. Degree of agreement and disagreement b. Compliance to a. 6 hours b. 4 hours c. 3 hours d. 2 hours
expected standards c. Level of satisfaction d. Degree of acceptance
98.Nurse Monique is monitoring the effectiveness of a client's drug a. Ventilator assistance b. CVP readings c. EKG tracings d. Continuous
therapy. When should the nurse Monique obtain a blood sample to CPR
measure the trough drug level? 10. A trial for vaginal delivery after an earlier caesareans, would
a. 1 hour before administering the next dose. b. Immediately before likely to be given to a gravida, who had:
administering the next dose. c. Immediately after administering the a. First low transverse cesarean was for active herpes type 2
next dose. d. 30 minutes after administering the next dose. infections; vaginal culture at 39 weeks pregnancy was positive. b.
99.Nurse May is aware that the main advantage of using a floor First and second caesareans were for cephalopelvic disproportion. c.
stock system is: First caesarean through a classic incision as a result of severe fetal
a. The nurse can implement medication orders quickly. b. The nurse distress. d. First low transverse caesarean was for breech position.
receives input from the pharmacist. c. The system minimizes Fetus in this pregnancy is in a vertex presentation.
transcription errors. d. The system reinforces accurate calculations. 11.Nurse Ryan is aware that the best initial approach when trying to
100. Nurse Oliver is assessing a client's abdomen. Which finding take a crying toddler’s temperature is:
should the nurse report as abnormal? a. Talk to the mother first and then to the toddler. b. Bring extra
a. Dullness over the liver. b. Bowel sounds occurring every 10 help so it can be done quickly. c. Encourage the mother to hold the
seconds. c. Shifting dullness over the abdomen. d. Vascular sounds child. d. Ignore the crying and screaming.
heard over the renal arteries. 12.Baby Tina a 3 month old infant just had a cleft lip and palate
Nursing Crib – Student Nurses’ Community 25 repair. What should the nurse do to prevent trauma to operative
site?
NURSING PRACTICE II a. Avoid touching the suture line, even when cleaning. b. Place the
Community Health Nursing and Care of the Mother and Child baby in prone position. c. Give the baby a pacifier. d. Place the
infant’s arms in soft elbow restraints.
TEST II - Community Health Nursing and Care of the Mother and 13.Which action should nurse Marian include in the care plan for a 2
Child month old with heart failure?
1. May arrives at the health care clinic and tells the nurse that her a. Feed the infant when he cries. b. Allow the infant to rest before
last menstrual period was 9 weeks ago. She also tells the nurse that feeding. c. Bathe the infant and administer medications before
a home pregnancy test was positive but she began to have mild feeding. d. Weigh and bathe the infant before feeding.
cramps and is now having moderate vaginal bleeding. During the 14.Nurse Hazel is teaching a mother who plans to discontinue breast
physical examination of the client, the nurse notes that May has a feeding after 5 months. The nurse should advise her to include
dilated cervix. The nurse determines that May is experiencing which which foods in her infant’s diet?
type of abortion? a. Skim milk and baby food. b. Whole milk and baby food. c. Iron-rich
a. Inevitable b. Incomplete c. Threatened d. Septic formula only. d. Iron-rich formula and baby food.
2. Nurse Reese is reviewing the record of a pregnant client for her 15.Mommy Linda is playing with her infant, who is sitting securely
first prenatal visit. Which of the following data, if noted on the alone on the floor of the clinic. The mother hides a toy behind her
client’s record, would alert the nurse that the client is at risk for a back and the
spontaneous abortion? Nursing Crib – Student Nurses’ Community 29
a. Age 36 years b. History of syphilis c. History of genital herpes d. infant looks for it. The nurse is aware that estimated age of the
History of diabetes mellitus infant would be:
3. Nurse Hazel is preparing to care for a client who is newly admitted a. 6 months b. 4 months c. 8 months d. 10 months
to the hospital with a possible diagnosis of ectopic pregnancy. Nurse 16.Which of the following is the most prominent feature of public
Hazel develops a plan of care for the client and determines that health nursing?
which of the following nursing actions is the priority? a. It involves providing home care to sick people who are not
a. Monitoring weight b. Assessing for edema c. Monitoring apical confined in the hospital. b. Services are provided free of charge to
pulse d. Monitoring temperature people within the catchments area. c. The public health nurse
4. Nurse Oliver is teaching a diabetic pregnant client about nutrition functions as part of a team providing a public health nursing
and insulin needs during pregnancy. The nurse determines that the services. d. Public health nursing focuses on preventive, not curative,
client understands dietary and insulin needs if the client states that services.
the second half of pregnancy require: 17.When the nurse determines whether resources were maximized
a. Decreased caloric intake b. Increased caloric intake c. Decreased in implementing Ligtas Tigdas, she is evaluating
Insulin d. Increase Insulin a. Effectiveness b. Efficiency c. Adequacy d. Appropriateness
Nursing Crib – Student Nurses’ Community 27 18.Vangie is a new B.S.N. graduate. She wants to become a Public
5. Nurse Michelle is assessing a 24 year old client with a diagnosis of Health Nurse. Where should she apply?
hydatidiform mole. She is aware that one of the following is a. Department of Health b. Provincial Health Office c. Regional
unassociated with this condition? Health Office d. Rural Health Unit
a. Excessive fetal activity. b. Larger than normal uterus for 19.Tony is aware the Chairman of the Municipal Health Board is:
gestational age. c. Vaginal bleeding d. Elevated levels of human a. Mayor b. Municipal Health Officer c. Public Health Nurse d. Any
chorionic gonadotropin. qualified physician
6. A pregnant client is receiving magnesium sulfate for severe 20.Myra is the public health nurse in a municipality with a total
pregnancy induced hypertension (PIH). The clinical findings that population of about 20,000. There are 3 rural health midwives
would warrant use of the antidote , calcium gluconate is: among the RHU personnel. How many more midwife items will the
a. Urinary output 90 cc in 2 hours. b. Absent patellar reflexes. c. RHU need?
Rapid respiratory rate above 40/min. d. Rapid rise in blood pressure. Nursing Crib – Student Nurses’ Community 30
7. During vaginal examination of Janah who is in labor, the a. 1 b. 2 c. 3 d. The RHU does not need any more midwife item.
presenting part is at station plus two. Nurse, correctly interprets it 21.According to Freeman and Heinrich, community health nursing is
as: a developmental service. Which of the following best illustrates this
a. Presenting part is 2 cm above the plane of the ischial spines. b. statement?
Biparietal diameter is at the level of the ischial spines. c. Presenting a. The community health nurse continuously develops himself
part in 2 cm below the plane of the ischial spines. d. Biparietal personally and professionally. b. Health education and community
diameter is 2 cm above the ischial spines. organizing are necessary in providing community health services. c.
8. A pregnant client is receiving oxytocin (Pitocin) for induction of Community health nursing is intended primarily for health
labor. A condition that warrant the nurse in-charge to discontinue promotion and prevention and treatment of disease. d. The goal of
I.V. infusion of Pitocin is: community health nursing is to provide nursing services to people in
a. Contractions every 1 ½ minutes lasting 70-80 seconds. b. Maternal their own places of residence.
temperature 101.2 c. Early decelerations in the fetal heart rate. d. 22.Nurse Tina is aware that the disease declared through
Fetal heart rate baseline 140-160 bpm. Presidential Proclamation No. 4 as a target for eradication in the
9. Calcium gluconate is being administered to a client with Philippines is?
pregnancy induced hypertension (PIH). A nursing action that must be a. Poliomyelitis b. Measles c. Rabies d. Neonatal tetanus
initiated as the plan of care throughout injection of the drug is: 23.May knows that the step in community organizing that involves
training of potential leaders in the community is:
a. Integration b. Community organization c. Community study d. a. “I should check the diaphragm carefully for holes every time I use
Core group formation it” b. “I may need a different size of diaphragm if I gain or lose
24.Beth a public health nurse takes an active role in community weight more than 20 pounds” c. “The diaphragm must be left in
participation. What is the primary goal of community organizing? place for atleast 6 hours after intercourse” d. “I really need to use
a. To educate the people regarding community health problems b. the diaphragm and jelly most during the middle of my menstrual
To mobilize the people to resolve community health problems c. To cycle”.
maximize the community’s resources in dealing with health 39.Hypoxia is a common complication of laryngotracheobronchitis.
problems. d. To maximize the community’s resources in dealing with Nurse Oliver should frequently assess a child with
health problems. laryngotracheobronchitis for:
Nursing Crib – Student Nurses’ Community 31 a. Drooling b. Muffled voice c. Restlessness d. Low-grade fever
25.Tertiary prevention is needed in which stage of the natural 40.How should Nurse Michelle guide a child who is blind to walk to
history of disease? the playroom?
a. Pre-pathogenesis b. Pathogenesis c. Prodromal d. Terminal a. Without touching the child, talk continuously as the child walks
26.The nurse is caring for a primigravid client in the labor and down the hall. b. Walk one step ahead, with the child’s hand on the
delivery area. Which condition would place the client at risk for nurse’s elbow. c. Walk slightly behind, gently guiding the child
disseminated intravascular coagulation (DIC)? forward. d. Walk next to the child, holding the child’s hand.
a. Intrauterine fetal death. b. Placenta accreta. c. Dysfunctional Nursing Crib – Student Nurses’ Community 34
labor. d. Premature rupture of the membranes. 41.When assessing a newborn diagnosed with ductus arteriosus,
27.A fullterm client is in labor. Nurse Betty is aware that the fetal Nurse Olivia should expect that the child most likely would have an:
heart rate would be: a. Loud, machinery-like murmur. b. Bluish color to the lips. c.
a. 80 to 100 beats/minute b. 100 to 120 beats/minute c. 120 to 160 Decreased BP reading in the upper extremities d. Increased BP
beats/minute d. 160 to 180 beats/minute reading in the upper extremities.
28.The skin in the diaper area of a 7 month old infant is excoriated 42.The reason nurse May keeps the neonate in a neutral thermal
and red. Nurse Hazel should instruct the mother to: environment is that when a newborn becomes too cool, the neonate
a. Change the diaper more often. b. Apply talc powder with diaper requires:
changes. c. Wash the area vigorously with each diaper change. d. a. Less oxygen, and the newborn’s metabolic rate increases. b. More
Decrease the infant’s fluid intake to decrease saturating diapers. oxygen, and the newborn’s metabolic rate decreases. c. More
29.Nurse Carla knows that the common cardiac anomalies in oxygen, and the newborn’s metabolic rate increases. d. Less oxygen,
children with Down Syndrome (tri-somy 21) is: and the newborn’s metabolic rate decreases.
a. Atrial septal defect b. Pulmonic stenosis c. Ventricular septal 43.Before adding potassium to an infant’s I.V. line, Nurse Ron must
defect d. Endocardial cushion defect be sure to assess whether this infant has:
30.Malou was diagnosed with severe preeclampsia is now receiving a. Stable blood pressure b. Patant fontanelles c. Moro’s reflex d.
I.V. magnesium sulfate. The adverse effects associated with Voided
magnesium sulfate is: 44.Nurse Carla should know that the most common causative factor
a. Anemia of dermatitis in infants and younger children is:
Nursing Crib – Student Nurses’ Community 32 a. Baby oil b. Baby lotion c. Laundry detergent d. Powder with
b. Decreased urine output c. Hyperreflexia d. Increased respiratory cornstarch
rate 45.During tube feeding, how far above an infant’s stomach should
31.A 23 year old client is having her menstrual period every 2 weeks the nurse hold the syringe with formula?
that last for 1 week. This type of menstrual pattern is bets defined a. 6 inches b. 12 inches c. 18 inches d. 24 inches
by: 46.In a mothers’ class, Nurse Lhynnete discussed childhood diseases
a. Menorrhagia b. Metrorrhagia c. Dyspareunia d. Amenorrhea such as chicken pox. Which of the following statements about
32.Jannah is admitted to the labor and delivery unit. The critical chicken pox is correct?
laboratory result for this client would be: Nursing Crib – Student Nurses’ Community 35
a. Oxygen saturation b. Iron binding capacity c. Blood typing d. a. The older one gets, the more susceptible he becomes to the
Serum Calcium complications of chicken pox. b. A single attack of chicken pox will
33.Nurse Gina is aware that the most common condition found prevent future episodes, including conditions such as shingles. c. To
during the second-trimester of pregnancy is: prevent an outbreak in the community, quarantine may be imposed
a. Metabolic alkalosis b. Respiratory acidosis c. Mastitis d. by health authorities. d. Chicken pox vaccine is best given when
Physiologic anemia there is an impending outbreak in the community.
34.Nurse Lynette is working in the triage area of an emergency 47.Barangay Pinoy had an outbreak of German measles. To prevent
department. She sees that several pediatric clients arrive congenital rubella, what is the BEST advice that you can give to
simultaneously. The client who needs to be treated first is: women in the first trimester of pregnancy in the barangay Pinoy?
a. A crying 5 year old child with a laceration on his scalp. b. A 4 year a. Advice them on the signs of German measles. b. Avoid crowded
old child with a barking coughs and flushed appearance. c. A 3 year places, such as markets and movie houses. c. Consult at the health
old child with Down syndrome who is pale and asleep in his mother’s center where rubella vaccine may be given. d. Consult a physician
arms. d. A 2 year old infant with stridorous breath sounds, sitting up who may give them rubella immunoglobulin.
in his mother’s arms and drooling. 48.Myrna a public health nurse knows that to determine possible
35.Maureen in her third trimester arrives at the emergency room sources of sexually transmitted infections, the BEST method that
with painless vaginal bleeding. Which of the following conditions is may be undertaken is:
suspected? a. Contact tracing b. Community survey c. Mass screening tests d.
a. Placenta previa b. Abruptio placentae c. Premature labor d. Interview of suspects
Sexually transmitted disease 49.A 33-year old female client came for consultation at the health
Nursing Crib – Student Nurses’ Community 33 center with the chief complaint of fever for a week. Accompanying
36.A young child named Richard is suspected of having pinworms. symptoms were muscle pains and body malaise. A week after the
The community nurse collects a stool specimen to confirm the start of fever, the client noted yellowish discoloration of his sclera.
diagnosis. The nurse should schedule the collection of this specimen History showed that he waded in flood waters about 2 weeks before
for: the onset of symptoms. Based on her history, which disease
a. Just before bedtime b. After the child has been bathe c. Any time condition will you suspect?
during the day d. Early in the morning a. Hepatitis A b. Hepatitis B c. Tetanus d. Leptospirosis
37.In doing a child’s admission assessment, Nurse Betty should be 50.Mickey a 3-year old client was brought to the health center with
alert to note which signs or symptoms of chronic lead poisoning? the chief complaint of severe diarrhea and the passage of “rice
a. Irritability and seizures b. Dehydration and diarrhea c. Bradycardia water” stools. The client is most probably suffering from which
and hypotension d. Petechiae and hematuria condition?
38.To evaluate a woman’s understanding about the use of a. Giardiasis b. Cholera c. Amebiasis
diaphragm for family planning, Nurse Trish asks her to explain how Nursing Crib – Student Nurses’ Community 36
she will use the appliance. Which response indicates a need for d. Dysentery
further health teaching?
51.The most prevalent form of meningitis among children aged 2 Using the Integrated Management of Child Illness (IMCI) guidelines
months to 3 years is caused by which microorganism? of assessment, his breathing is considered as:
a. Hemophilus influenzae b. Morbillivirus c. Steptococcus a. Fast b. Slow c. Normal d. Insignificant
pneumoniae d. Neisseria meningitidis Nursing Crib – Student Nurses’ Community 39
52.The student nurse is aware that the pathognomonic sign of 66.Maylene had just received her 4th dose of tetanus toxoid. She is
measles is Koplik’s spot and you may see Koplik’s spot by inspecting aware that her baby will have protection against tetanus for
the: a. 1 year b. 3 years c. 5 years d. Lifetime
a. Nasal mucosa b. Buccal mucosa c. Skin on the abdomen d. Skin on 67.Nurse Ron is aware that unused BCG should be discarded after
neck how many hours of reconstitution?
53.Angel was diagnosed as having Dengue fever. You will say that a. 2 hours b. 4 hours c. 8 hours d. At the end of the day
there is slow capillary refill when the color of the nailbed that you 68.The nurse explains to a breastfeeding mother that breast milk is
pressed does not return within how many seconds? sufficient for all of the baby’s nutrient needs only up to:
a. 3 seconds b. 6 seconds c. 9 seconds d. 10 seconds a. 5 months b. 6 months c. 1 year d. 2 years
54.In Integrated Management of Childhood Illness, the nurse is 69.Nurse Ron is aware that the gestational age of a conceptus that is
aware that the severe conditions generally require urgent referral to considered viable (able to live outside the womb) is:
a hospital. Which of the following severe conditions DOES NOT a. 8 weeks b. 12 weeks c. 24 weeks d. 32 weeks
always require urgent referral to a hospital? 70.When teaching parents of a neonate the proper position for the
a. Mastoiditis b. Severe dehydration c. Severe pneumonia d. Severe neonate’s sleep, the nurse Patricia stresses the importance of
febrile disease placing the neonate on his back to reduce the risk of which of the
55.Myrna a public health nurse will conduct outreach immunization following?
in a barangay Masay with a population of about 1500. The estimated a. Aspiration b. Sudden infant death syndrome (SIDS) c. Suffocation
number of infants in the barangay would be: d. Gastroesophageal reflux (GER)
a. 45 infants b. 50 infants c. 55 infants d. 65 infants 71.Which finding might be seen in baby James a neonate suspected
Nursing Crib – Student Nurses’ Community 37 of having an infection?
56.The community nurse is aware that the biological used in a. Flushed cheeks b. Increased temperature
Expanded Program on Immunization (EPI) should NOT be stored in Nursing Crib – Student Nurses’ Community 40
the freezer? c. Decreased temperature d. Increased activity level
a. DPT b. Oral polio vaccine c. Measles vaccine d. MMR 72.Baby Jenny who is small-for-gestation is at increased risk during
57.It is the most effective way of controlling schistosomiasis in an the transitional period for which complication?
endemic area? a. Anemia probably due to chronic fetal hyposia b. Hyperthermia
a. Use of molluscicides b. Building of foot bridges c. Proper use of due to decreased glycogen stores c. Hyperglycemia due to decreased
sanitary toilets d. Use of protective footwear, such as rubber boots glycogen stores d. Polycythemia probably due to chronic fetal
58.Several clients is newly admitted and diagnosed with leprosy. hypoxia
Which of the following clients should be classified as a case of 73.Marjorie has just given birth at 42 weeks’ gestation. When the
multibacillary leprosy? nurse assessing the neonate, which physical finding is expected?
a. 3 skin lesions, negative slit skin smear b. 3 skin lesions, positive slit a. A sleepy, lethargic baby b. Lanugo covering the body c.
skin smear c. 5 skin lesions, negative slit skin smear d. 5 skin lesions, Desquamation of the epidermis d. Vernix caseosa covering the body
positive slit skin smear 74.After reviewing the Myrna’s maternal history of magnesium
59.Nurses are aware that diagnosis of leprosy is highly dependent on sulfate during labor, which condition would nurse Richard anticipate
recognition of symptoms. Which of the following is an early sign of as a potential problem in the neonate?
leprosy? a. Hypoglycemia b. Jitteriness c. Respiratory depression d.
a. Macular lesions b. Inability to close eyelids c. Thickened painful Tachycardia
nerves d. Sinking of the nosebridge 75.Which symptom would indicate the Baby Alexandra was adapting
60.Marie brought her 10 month old infant for consultation because appropriately to extra-uterine life without difficulty?
of fever, started 4 days prior to consultation. In determining malaria a. Nasal flaring b. Light audible grunting c. Respiratory rate 40 to 60
risk, what will you do? breaths/minute d. Respiratory rate 60 to 80 breaths/minute
a. Perform a tourniquet test. b. Ask where the family resides. c. Get 76. When teaching umbilical cord care for Jennifer a new mother,
a specimen for blood smear. d. Ask if the fever is present everyday. the nurse Jenny would include which information?
61.Susie brought her 4 years old daughter to the RHU because of a. Apply peroxide to the cord with each diaper change b. Cover the
cough and colds. Following the IMCI assessment guide, which of the cord with petroleum jelly after bathing c. Keep the cord dry and
following is a danger sign that indicates the need for urgent referral open to air d. Wash the cord with soap and water each day during a
to a hospital? tub bath.
Nursing Crib – Student Nurses’ Community 38 77.Nurse John is performing an assessment on a neonate. Which of
a. Inability to drink b. High grade fever c. Signs of severe dehydration the following findings is considered common in the healthy
d. Cough for more than 30 days neonate?
62.Jimmy a 2-year old child revealed “baggy pants”. As a nurse, using Nursing Crib – Student Nurses’ Community 41
the IMCI guidelines, how will you manage Jimmy? a. Simian crease b. Conjunctival hemorrhage c. Cystic hygroma d.
a. Refer the child urgently to a hospital for confinement. b. Bulging fontanelle
Coordinate with the social worker to enroll the child in a feeding 78.Dr. Esteves decides to artificially rupture the membranes of a
program. c. Make a teaching plan for the mother, focusing on menu mother who is on labor. Following this procedure, the nurse Hazel
planning for her child. d. Assess and treat the child for health checks the fetal heart tones for which the following reasons?
problems like infections and intestinal parasitism. a. To determine fetal well-being. b. To assess for prolapsed cord c.
63.Gina is using Oresol in the management of diarrhea of her 3-year To assess fetal position d. To prepare for an imminent delivery.
old child. She asked you what to do if her child vomits. As a nurse 79.Which of the following would be least likely to indicate
you will tell her to: anticipated bonding behaviors by new parents?
a. Bring the child to the nearest hospital for further assessment. b. a. The parents’ willingness to touch and hold the new born. b. The
Bring the child to the health center for intravenous fluid therapy. c. parent’s expression of interest about the size of the new born. c. The
Bring the child to the health center for assessment by the physician. parents’ indication that they want to see the newborn. d. The
d. Let the child rest for 10 minutes then continue giving Oresol more parents’ interactions with each other.
slowly. 80.Following a precipitous delivery, examination of the client's
64.Nikki a 5-month old infant was brought by his mother to the vagina reveals a fourth-degree laceration. Which of the following
health center because of diarrhea for 4 to 5 times a day. Her skin would be contraindicated when caring for this client?
goes back slowly after a skin pinch and her eyes are sunken. Using a. Applying cold to limit edema during the first 12 to 24 hours. b.
the IMCI guidelines, you will classify this infant in which category? Instructing the client to use two or more peripads to cushion the
a. No signs of dehydration b. Some dehydration c. Severe area. c. Instructing the client on the use of sitz baths if ordered. d.
dehydration d. The data is insufficient. Instructing the client about the importance of perineal (kegel)
65.Chris a 4-month old infant was brought by her mother to the exercises.
health center because of cough. His respiratory rate is 42/minute.
81. A pregnant woman accompanied by her husband, seeks appears in 6 to 12 hours. d. A flat circumcised area over 10 mm in
admission to the labor and delivery area. She states that she's in diameter appears in 48 to 72 hours.
labor and says she attended the facility clinic for prenatal care. 95. Dianne, 24 year-old is 27 weeks’ pregnant arrives at her
Which question should the nurse Oliver ask her first? physician’s office with complaints of fever, nausea, vomiting,
a. “Do you have any chronic illnesses?” b. “Do you have any malaise, unilateral flank pain, and costovertebral angle tenderness.
allergies?” c. “What is your expected due date?” d. “Who will be Which of the following diagnoses is most likely?
with you during labor?” a. Asymptomatic bacteriuria b. Bacterial vaginosis c. Pyelonephritis
82.A neonate begins to gag and turns a dusky color. What should the d. Urinary tract infection (UTI)
nurse do first? 96. Rh isoimmunization in a pregnant client develops during which
Nursing Crib – Student Nurses’ Community 42 of the following conditions?
a. Calm the neonate. b. Notify the physician. c. Provide oxygen via a. Rh-positive maternal blood crosses into fetal blood, stimulating
face mask as ordered d. Aspirate the neonate’s nose and mouth with fetal antibodies. b. Rh-positive fetal blood crosses into maternal
a bulb syringe. blood, stimulating maternal antibodies. c. Rh-negative fetal blood
83. When a client states that her "water broke," which of the crosses into maternal blood, stimulating maternal antibodies. d. Rh-
following actions would be inappropriate for the nurse to do? negative maternal blood crosses into fetal blood, stimulating fetal
a. Observing the pooling of straw-colored fluid. b. Checking vaginal antibodies.
discharge with nitrazine paper. c. Conducting a bedside ultrasound 97. To promote comfort during labor, the nurse John advises a client
for an amniotic fluid index. d. Observing for flakes of vernix in the to assume certain positions and avoid others. Which position may
vaginal discharge. cause maternal hypotension and fetal hypoxia?
84. A baby girl is born 8 weeks premature. At birth, she has no a. Lateral position b. Squatting position c. Supine position
spontaneous respirations but is successfully resuscitated. Within Nursing Crib – Student Nurses’ Community 45
several hours she develops respiratory grunting, cyanosis, d. Standing position
tachypnea, nasal flaring, and retractions. She's diagnosed with 98. Celeste who used heroin during her pregnancy delivers a
respiratory distress syndrome, intubated, and placed on a ventilator. neonate. When assessing the neonate, the nurse Lhynnette expects
Which nursing action should be included in the baby's plan of care to to find:
prevent retinopathy of prematurity? a. Lethargy 2 days after birth. b. Irritability and poor sucking. c. A
a. Cover his eyes while receiving oxygen. b. Keep her body flattened nose, small eyes, and thin lips. d. Congenital defects such
temperature low. c. Monitor partial pressure of oxygen (Pao2) as limb anomalies.
levels. d. Humidify the oxygen. 99. The uterus returns to the pelvic cavity in which of the following
85. Which of the following is normal newborn calorie intake? time frames?
a. 110 to 130 calories per kg. b. 30 to 40 calories per lb of body a. 7th to 9th day postpartum. b. 2 weeks postpartum. c. End of 6th
weight. c. At least 2 ml per feeding d. 90 to 100 calories per kg week postpartum. d. When the lochia changes to alba.
86. Nurse John is knowledgeable that usually individual twins will 100. Maureen, a primigravida client, age 20, has just completed a
grow appropriately and at the same rate as singletons until how difficult, forceps-assisted delivery of twins. Her labor was unusually
many weeks? long and required oxytocin (Pitocin) augmentation. The nurse who's
a. 16 to 18 weeks b. 18 to 22 weeks c. 30 to 32 weeks d. 38 to 40 caring for her should stay alert for:
weeks a. Uterine inversion b. Uterine atony c. Uterine involution d. Uterine
87. Which of the following classifications applies to monozygotic discomfort
twins for whom the cleavage of the fertilized ovum occurs more Nursing Crib – Student Nurses’ Community 46
than 13 days after fertilization?
a. conjoined twins b. diamniotic dichorionic twins NURSING PRACTICE III
Nursing Crib – Student Nurses’ Community 43 Care of Clients with Physiologic and Psychosocial Alterations
c. diamniotic monochorionic twin d. monoamniotic monochorionic Nursing Crib – Student Nurses’ Community 47
twins TEST III - Care of Clients with Physiologic and Psychosocial
88. Tyra experienced painless vaginal bleeding has just been Alterations
diagnosed as having a placenta previa. Which of the following 1. Nurse Michelle should know that the drainage is normal 4 days
procedures is usually performed to diagnose placenta previa? after a sigmoid colostomy when the stool is:
a. Amniocentesis b. Digital or speculum examination c. External fetal a. Green liquid b. Solid formed c. Loose, bloody d. Semiformed
monitoring d. Ultrasound 2. Where would nurse Kristine place the call light for a male client
89. Nurse Arnold knows that the following changes in respiratory with a right-sided brain attack and left homonymous hemianopsia?
functioning during pregnancy is considered normal: a. On the client’s right side b. On the client’s left side c. Directly in
a. Increased tidal volume b. Increased expiratory volume c. front of the client d. Where the client like
Decreased inspiratory capacity d. Decreased oxygen consumption 3. A male client is admitted to the emergency department following
90. Emily has gestational diabetes and it is usually managed by an accident. What are the first nursing actions of the nurse?
which of the following therapy? a. Check respiration, circulation, neurological response. b. Align the
a. Diet b. Long-acting insulin c. Oral hypoglycemic d. Oral spine, check pupils, and check for hemorrhage. c. Check respirations,
hypoglycemic drug and insulin stabilize spine, and check circulation. d. Assess level of
91. Magnesium sulfate is given to Jemma with preeclampsia to consciousness and circulation.
prevent which of the following condition? 4. In evaluating the effect of nitroglycerin, Nurse Arthur should know
a. Hemorrhage b. Hypertension c. Hypomagnesemia d. Seizure that it reduces preload and relieves angina by:
92. Cammile with sickle cell anemia has an increased risk for having a. Increasing contractility and slowing heart rate. b. Increasing AV
a sickle cell crisis during pregnancy. Aggressive management of a conduction and heart rate. c. Decreasing contractility and oxygen
sickle cell crisis includes which of the following measures? consumption. d. Decreasing venous return through vasodilation.
a. Antihypertensive agents b. Diuretic agents c. I.V. fluids d. 5. Nurse Patricia finds a female client who is post-myocardial
Acetaminophen (Tylenol) for pain infarction (MI) slumped on the side rails of the bed and
Nursing Crib – Student Nurses’ Community 44 unresponsive to shaking or shouting. Which is the nurse next action?
93. Which of the following drugs is the antidote for magnesium a. Call for help and note the time. b. Clear the airway c. Give two
toxicity? sharp thumps to the precordium, and check the pulse. d. Administer
a. Calcium gluconate (Kalcinate) b. Hydralazine (Apresoline) c. two quick blows.
Naloxone (Narcan) d. Rho (D) immune globulin (RhoGAM) 6. Nurse Monett is caring for a client recovering from gastro-
94. Marlyn is screened for tuberculosis during her first prenatal visit. intestinal bleeding. The nurse should:
An intradermal injection of purified protein derivative (PPD) of the Nursing Crib – Student Nurses’ Community 48
tuberculin bacilli is given. She is considered to have a positive test a. Plan care so the client can receive 8 hours of uninterrupted sleep
for which of the following results? each night. b. Monitor vital signs every 2 hours. c. Make sure that
a. An indurated wheal under 10 mm in diameter appears in 6 to 12 the client takes food and medications at prescribed intervals. d.
hours. b. An indurated wheal over 10 mm in diameter appears in 48 Provide milk every 2 to 3 hours.
to 72 hours. c. A flat circumcised area under 10 mm in diameter 7. A male client was on warfarin (Coumadin) before admission, and
has been receiving heparin I.V. for 2 days. The partial
thromboplastin time (PTT) is 68 seconds. What should Nurse Carla a. Cancerous lumps b. Areas of thickness or fullness c. Changes from
do? previous examinations. d. Fibrocystic masses
a. Stop the I.V. infusion of heparin and notify the physician. b. 21. When caring for a female client who is being treated for
Continue treatment as ordered. c. Expect the warfarin to increase hyperthyroidism, it is important to:
the PTT. d. Increase the dosage, because the level is lower than a. Provide extra blankets and clothing to keep the client warm. b.
normal. Monitor the client for signs of restlessness, sweating, and excessive
8. A client undergone ileostomy, when should the drainage weight loss during thyroid replacement therapy. c. Balance the
appliance be applied to the stoma? client’s periods of activity and rest. d. Encourage the client to be
a. 24 hours later, when edema has subsided. b. In the operating active to prevent constipation.
room. c. After the ileostomy begin to function. d. When the client is Nursing Crib – Student Nurses’ Community 51
able to begin self-care procedures. 22. Nurse Kris is teaching a client with history of atherosclerosis. To
9. A client undergone spinal anesthetic, it will be important that the decrease the risk of atherosclerosis, the nurse should encourage the
nurse immediately position the client in: client to:
a. On the side, to prevent obstruction of airway by tongue. b. Flat on a. Avoid focusing on his weight. b. Increase his activity level. c.
back. c. On the back, with knees flexed 15 degrees. d. Flat on the Follow a regular diet. d. Continue leading a high-stress lifestyle.
stomach, with the head turned to the side. 23. Nurse Greta is working on a surgical floor. Nurse Greta must
10.While monitoring a male client several hours after a motor logroll a client following a:
vehicle accident, which assessment data suggest increasing a. Laminectomy b. Thoracotomy c. Hemorrhoidectomy d.
intracranial pressure? Cystectomy.
a. Blood pressure is decreased from 160/90 to 110/70. b. Pulse is 24. A 55-year old client underwent cataract removal with intraocular
increased from 87 to 95, with an occasional skipped beat. c. The lens implant. Nurse Oliver is giving the client discharge instructions.
client is oriented when aroused from sleep, and goes back to sleep These instructions should include which of the following?
immediately. d. The client refuses dinner because of anorexia. a. Avoid lifting objects weighing more than 5 lb (2.25 kg). b. Lie on
11.Mrs. Cruz, 80 years old is diagnosed with pneumonia. Which of your abdomen when in bed c. Keep rooms brightly lit. d. Avoiding
the following symptoms may appear first? straining during bowel movement or bending at the waist.
a. Altered mental status and dehydration 25. George should be taught about testicular examinations during:
Nursing Crib – Student Nurses’ Community 49 a. when sexual activity starts b. After age 69 c. After age 40 d. Before
b. Fever and chills c. Hemoptysis and Dyspnea d. Pleuritic chest pain age 20.
and cough 26. A male client undergone a colon resection. While turning him,
12. A male client has active tuberculosis (TB). Which of the following wound dehiscence with evisceration occurs. Nurse Trish first
symptoms will be exhibit? response is to:
a. Chest and lower back pain b. Chills, fever, night sweats, and a. Call the physician b. Place a saline-soaked sterile dressing on the
hemoptysis c. Fever of more than 104°F (40°C) and nausea d. wound. c. Take a blood pressure and pulse. d. Pull the dehiscence
Headache and photophobia closed.
13. Mark, a 7-year-old client is brought to the emergency 27. Nurse Audrey is caring for a client who has suffered a severe
department. He’s tachypneic and afebrile and has a respiratory rate cerebrovascular accident. During routine assessment, the nurse
of 36 breaths/minute and has a nonproductive cough. He recently notices CheyneStrokes respirations. Cheyne-strokes respirations are:
had a cold. Form this history; the client may have which of the a. A progressively deeper breaths followed by shallower breaths
following conditions? with apneic periods.
a. Acute asthma b. Bronchial pneumonia c. Chronic obstructive Nursing Crib – Student Nurses’ Community 52
pulmonary disease (COPD) d. Emphysema b. Rapid, deep breathing with abrupt pauses between each breath.
14. Marichu was given morphine sulfate for pain. She is sleeping and c. Rapid, deep breathing and irregular breathing without pauses. d.
her respiratory rate is 4 breaths/minute. If action isn’t taken quickly, Shallow breathing with an increased respiratory rate.
she might have which of the following reactions? 28. Nurse Bea is assessing a male client with heart failure. The
a. Asthma attack b. Respiratory arrest c. Seizure d. Wake up on his breath sounds commonly auscultated in clients with heart failure
own are:
15. A 77-year-old male client is admitted for elective knee surgery. a. Tracheal b. Fine crackles c. Coarse crackles d. Friction rubs
Physical examination reveals shallow respirations but no sign of 29. The nurse is caring for Kenneth experiencing an acute asthma
respiratory distress. Which of the following is a normal physiologic attack. The client stops wheezing and breath sounds aren’t audible.
change related to aging? The reason for this change is that:
a. Increased elastic recoil of the lungs b. Increased number of a. The attack is over. b. The airways are so swollen that no air cannot
functional capillaries in the alveoli c. Decreased residual volume d. get through. c. The swelling has decreased. d. Crackles have replaced
Decreased vital capacity wheezes.
16. Nurse John is caring for a male client receiving lidocaine I.V. 30. Mike with epilepsy is having a seizure. During the active seizure
Which factor is the most relevant to administration of this phase, the nurse should:
medication? a. Place the client on his back remove dangerous objects, and insert
a. Decrease in arterial oxygen saturation (SaO2) when measured a bite block. b. Place the client on his side, remove dangerous
with a pulse oximeter. b. Increase in systemic blood pressure. objects, and insert a bite block. c. Place the client o his back, remove
Nursing Crib – Student Nurses’ Community 50 dangerous objects, and hold down his arms. d. Place the client on his
c. Presence of premature ventricular contractions (PVCs) on a side, remove dangerous objects, and protect his head.
cardiac monitor. d. Increase in intracranial pressure (ICP). 31. After insertion of a cheat tube for a pneumothorax, a client
17. Nurse Ron is caring for a male client taking an anticoagulant. The becomes hypotensive with neck vein distention, tracheal shift,
nurse should teach the client to: absent breath sounds, and diaphoresis. Nurse Amanda suspects a
a. Report incidents of diarrhea. b. Avoid foods high in vitamin K c. tension pneumothorax has occurred. What cause of tension
Use a straight razor when shaving. d. Take aspirin to pain relief. pneumothorax should the nurse check for?
18. Nurse Lhynnette is preparing a site for the insertion of an I.V. a. Infection of the lung. b. Kinked or obstructed chest tube c.
catheter. The nurse should treat excess hair at the site by: Excessive water in the water-seal chamber d. Excessive chest tube
a. Leaving the hair intact b. Shaving the area c. Clipping the hair in drainage
the area d. Removing the hair with a depilatory. 32. Nurse Maureen is talking to a male client, the client begins
19. Nurse Michelle is caring for an elderly female with osteoporosis. choking on his lunch. He’s coughing forcefully. The nurse should:
When teaching the client, the nurse should include information Nursing Crib – Student Nurses’ Community 53
about which major complication: a. Stand him up and perform the abdominal thrust maneuver from
a. Bone fracture b. Loss of estrogen c. Negative calcium balance d. behind. b. Lay him down, straddle him, and perform the abdominal
Dowager’s hump thrust maneuver. c. Leave him to get assistance d. Stay with him but
20. Nurse Len is teaching a group of women to perform BSE. The not intervene at this time.
nurse should explain that the purpose of performing the 33. Nurse Ron is taking a health history of an 84 year old client.
examination is to discover: Which information will be most useful to the nurse for planning
care?
a. General health for the last 10 years. b. Current health promotion a. Explain the risks of not having the surgery b. Notifying the
activities. c. Family history of diseases. d. Marital status. physician immediately c. Notifying the nursing supervisor d.
34. When performing oral care on a comatose client, Nurse Krina Recording the client’s refusal in the nurses’ notes
should: 45. During the endorsement, which of the following clients should
a. Apply lemon glycerin to the client’s lips at least every 2 hours. b. the on-duty nurse assess first?
Brush the teeth with client lying supine. c. Place the client in a side a. The 58-year-old client who was admitted 2 days ago with heart
lying position, with the head of the bed lowered. d. Clean the client’s failure, blood pressure of 126/76 mm Hg, and a respiratory rate of
mouth with hydrogen peroxide. 22 breaths/ minute. b. The 89-year-old client with end-stage right-
35. A 77-year-old male client is admitted with a diagnosis of sided heart failure, blood pressure of 78/50 mm Hg, and a “do not
dehydration and change in mental status. He’s being hydrated with resuscitate” order c. The 62-year-old client who was admitted 1 day
L.V. fluids. When the nurse takes his vital signs, she notes he has a ago with thrombophlebitis and is receiving L.V. heparin d. The 75-
fever of 103°F (39.4°C) a cough producing yellow sputum and year-old client who was admitted 1 hour ago with new-onset atrial
pleuritic chest pain. The nurse suspects this client may have which of fibrillation and is receiving L.V. dilitiazem (Cardizem)
the following conditions? 46. Honey, a 23-year old client complains of substernal chest pain
a. Adult respiratory distress syndrome (ARDS) b. Myocardial and states that her heart feels like “it’s racing out of the chest”. She
infarction (MI) c. Pneumonia d. Tuberculosis reports no history of cardiac disorders. The nurse attaches her to a
36. Nurse Oliver is working in a out patient clinic. He has been cardiac monitor and notes sinus tachycardia with a rate of
alerted that there is an outbreak of tuberculosis (TB). Which of the 136beats/minutes. Breath sounds are clear and the respiratory rate
following clients entering the clinic today most likely to have TB? is 26 breaths/minutes. Which of the following drugs should the
a. A 16-year-old female high school student b. A 33-year-old day- nurse question the client about using?
care worker c. A 43-yesr-old homeless man with a history of a. Barbiturates
alcoholism d. A 54-year-old businessman Nursing Crib – Student Nurses’ Community 56
37. Virgie with a positive Mantoux test result will be sent for a chest b. Opioids c. Cocaine d. Benzodiazepines
X-ray. The nurse is aware that which of the following reasons this is 47. A 51-year-old female client tells the nurse in-charge that she has
done? found a painless lump in her right breast during her monthly self-
a. To confirm the diagnosis examination. Which assessment finding would strongly suggest that
Nursing Crib – Student Nurses’ Community 54 this client's lump is cancerous?
b. To determine if a repeat skin test is needed c. To determine the a. Eversion of the right nipple and mobile mass b. Nonmobile mass
extent of lesions d. To determine if this is a primary or secondary with irregular edges c. Mobile mass that is soft and easily delineated
infection d. Nonpalpable right axillary lymph nodes
38. Kennedy with acute asthma showing inspiratory and expiratory 48. A 35-year-old client with vaginal cancer asks the nurse, "What is
wheezes and a decreased forced expiratory volume should be the usual treatment for this type of cancer?" Which treatment
treated with which of the following classes of medication right should the nurse name?
away? a. Surgery b. Chemotherapy c. Radiation d. Immunotherapy
a. Beta-adrenergic blockers b. Bronchodilators c. Inhaled steroids d. 49. Cristina undergoes a biopsy of a suspicious lesion. The biopsy
Oral steroids report classifies the lesion according to the TNM staging system as
39. Mr. Vasquez 56-year-old client with a 40-year history of smoking follows: TIS, N0, M0. What does this classification mean?
one to two packs of cigarettes per day has a chronic cough a. No evidence of primary tumor, no abnormal regional lymph
producing thick sputum, peripheral edema and cyanotic nail beds. nodes, and no evidence of distant metastasis b. Carcinoma in situ,
Based on this information, he most likely has which of the following no abnormal regional lymph nodes, and no evidence of distant
conditions? metastasis c. Can't assess tumor or regional lymph nodes and no
a. Adult respiratory distress syndrome (ARDS) b. Asthma c. Chronic evidence of metastasis d. Carcinoma in situ, no demonstrable
obstructive bronchitis d. Emphysema metastasis of the regional lymph nodes, and ascending degrees of
Situation: Francis, age 46 is admitted to the hospital with diagnosis distant metastasis
of Chronic Lymphocytic Leukemia. 50. Lydia undergoes a laryngectomy to treat laryngeal cancer. When
40. The treatment for patients with leukemia is bone marrow teaching the client how to care for the neck stoma, the nurse should
transplantation. Which statement about bone marrow include which instruction?
transplantation is not correct? a. "Keep the stoma uncovered." b. "Keep the stoma dry." c. "Have a
a. The patient is under local anesthesia during the procedure b. The family member perform stoma care initially until you get used to the
aspirated bone marrow is mixed with heparin. c. The aspiration site procedure." d. "Keep the stoma moist."
is the posterior or anterior iliac crest. d. The recipient receives Nursing Crib – Student Nurses’ Community 57
cyclophosphamide (Cytoxan) for 4 consecutive days before the 51. A 37-year-old client with uterine cancer asks the nurse, "Which is
procedure. the most common type of cancer in women?" The nurse replies that
41. After several days of admission, Francis becomes disoriented and it's breast cancer. Which type of cancer causes the most deaths in
complains of frequent headaches. The nurse in-charge first action women?
would be: a. Breast cancer b. Lung cancer c. Brain cancer d. Colon and rectal
a. Call the physician b. Document the patient’s status in his charts. c. cancer
Prepare oxygen treatment d. Raise the side rails 52. Antonio with lung cancer develops Horner's syndrome when the
42. During routine care, Francis asks the nurse, “How can I be tumor invades the ribs and affects the sympathetic nerve ganglia.
anemic if this disease causes increased my white blood cell When assessing for signs and symptoms of this syndrome, the nurse
production?” The nurse in-charge best response would be that the should note:
increased number of white blood cells (WBC) is: a. miosis, partial eyelid ptosis, and anhidrosis on the affected side of
Nursing Crib – Student Nurses’ Community 55 the face. b. chest pain, dyspnea, cough, weight loss, and fever. c.
a. Crowd red blood cells b. Are not responsible for the anemia. c. arm and shoulder pain and atrophy of arm and hand muscles, both
Uses nutrients from other cells d. Have an abnormally short life span on the affected side. d. hoarseness and dysphagia.
of cells.
43. Diagnostic assessment of Francis would probably not reveal: 53. Vic asks the nurse what PSA is. The nurse should reply that it
a. Predominance of lymhoblasts b. Leukocytosis c. Abnormal blast stands for:
cells in the bone marrow d. Elevated thrombocyte counts a. prostate-specific antigen, which is used to screen for prostate
44. Robert, a 57-year-old client with acute arterial occlusion of the cancer. b. protein serum antigen, which is used to determine protein
left leg undergoes an emergency embolectomy. Six hours later, the levels. c. pneumococcal strep antigen, which is a bacteria that causes
nurse isn’t able to obtain pulses in his left foot using Doppler pneumonia. d. Papanicolaou-specific antigen, which is used to
ultrasound. The nurse immediately notifies the physician, and asks screen for cervical cancer.
her to prepare the client for surgery. As the nurse enters the client’s 54. What is the most important postoperative instruction that nurse
room to prepare him, he states that he won’t have any more Kate must give a client who has just returned from the operating
surgery. Which of the following is the best initial response by the room after receiving a subarachnoid block?
nurse? a. "Avoid drinking liquids until the gag reflex returns." b. "Avoid
eating milk products for 24 hours." c. "Notify a nurse if you
experience blood in your urine." d. "Remain supine for the time 68. Nurse Len should expect to administer which medication to a
specified by the physician." client with gout?
55. A male client suspected of having colorectal cancer will require a. aspirin b. furosemide (Lasix) c. colchicines d. calcium gluconate
which diagnostic study to confirm the diagnosis? (Kalcinate)
a. Stool Hematest b. Carcinoembryonic antigen (CEA) c. 69. Mr. Domingo with a history of hypertension is diagnosed with
Sigmoidoscopy d. Abdominal computed tomography (CT) scan primary hyperaldosteronism. This diagnosis indicates that the
Nursing Crib – Student Nurses’ Community 58 client's hypertension is caused by excessive hormone secretion from
56. During a breast examination, which finding most strongly which of the following glands?
suggests that the Luz has breast cancer? a. Adrenal cortex b. Pancreas c. Adrenal medulla d. Parathyroid
a. Slight asymmetry of the breasts. b. A fixed nodular mass with 70. For a diabetic male client with a foot ulcer, the doctor orders bed
dimpling of the overlying skin c. Bloody discharge from the nipple d. rest, a wetto-dry dressing change every shift, and blood glucose
Multiple firm, round, freely movable masses that change with the monitoring before meals and bedtime. Why are wet-to-dry dressings
menstrual cycle used for this client?
57. A female client with cancer is being evaluated for possible a. They contain exudate and provide a moist wound environment. b.
metastasis. Which of the following is one of the most common They protect the wound from mechanical trauma and promote
metastasis sites for cancer cells? healing. c. They debride the wound and promote healing by
a. Liver b. Colon c. Reproductive tract d. White blood cells (WBCs) secondary intention. d. They prevent the entrance of
58. Nurse Mandy is preparing a client for magnetic resonance microorganisms and minimize wound discomfort.
imaging (MRI) to confirm or rule out a spinal cord lesion. During the 71. Nurse Zeny is caring for a client in acute addisonian crisis. Which
MRI scan, which of the following would pose a threat to the client? laboratory data would the nurse expect to find?
a. The client lies still. b. The client asks questions. c. The client hears a. Hyperkalemia
thumping sounds. d. The client wears a watch and wedding band. Nursing Crib – Student Nurses’ Community 61
59. Nurse Cecile is teaching a female client about preventing b. Reduced blood urea nitrogen (BUN) c. Hypernatremia d.
osteoporosis. Which of the following teaching points is correct? Hyperglycemia
a. Obtaining an X-ray of the bones every 3 years is recommended to 72. A client is admitted for treatment of the syndrome of
detect bone loss. b. To avoid fractures, the client should avoid inappropriate antidiuretic hormone (SIADH). Which nursing
strenuous exercise. c. The recommended daily allowance of calcium intervention is appropriate?
may be found in a wide variety of foods. d. Obtaining the a. Infusing I.V. fluids rapidly as ordered b. Encouraging increased oral
recommended daily allowance of calcium requires taking a calcium intake c. Restricting fluids d. Administering glucose-containing I.V.
supplement. fluids as ordered
60. Before Jacob undergoes arthroscopy, the nurse reviews the 73. A female client tells nurse Nikki that she has been working hard
assessment findings for contraindications for this procedure. Which for the last 3 months to control her type 2 diabetes mellitus with
finding is a contraindication? diet and exercise. To determine the effectiveness of the client's
a. Joint pain b. Joint deformity c. Joint flexion of less than 50% d. efforts, the nurse should check:
Joint stiffness a. urine glucose level. b. fasting blood glucose level. c. serum
Nursing Crib – Student Nurses’ Community 59 fructosamine level. d. glycosylated hemoglobin level.
61. Mr. Rodriguez is admitted with severe pain in the knees. Which 74. Nurse Trinity administered neutral protamine Hagedorn (NPH)
form of arthritis is characterized by urate deposits and joint pain, insulin to a diabetic client at 7 a.m. At what time would the nurse
usually in the feet and legs, and occurs primarily in men over age 30? expect the client to be most at risk for a hypoglycemic reaction?
a. Septic arthritis b. Traumatic arthritis c. Intermittent arthritis d. a. 10:00 am b. Noon c. 4:00 pm d. 10:00 pm
Gouty arthritis 75. The adrenal cortex is responsible for producing which
62. A heparin infusion at 1,500 unit/hour is ordered for a 64-year-old substances?
client with stroke in evolution. The infusion contains 25,000 units of a. Glucocorticoids and androgens b. Catecholamines and
heparin in 500 ml of saline solution. How many milliliters per hour epinephrine c. Mineralocorticoids and catecholamines d.
should be given? Norepinephrine and epinephrine
a. 15 ml/hour b. 30 ml/hour c. 45 ml/hour d. 50 ml/hour 76. On the third day after a partial thyroidectomy, Proserfina
63. A 76-year-old male client had a thromboembolic right stroke; his exhibits muscle twitching and hyperirritability of the nervous
left arm is swollen. Which of the following conditions may cause system. When questioned, the client reports numbness and tingling
swelling after a stroke? of the mouth and fingertips. Suspecting a lifethreatening electrolyte
a. Elbow contracture secondary to spasticity b. Loss of muscle disturbance, the nurse notifies the surgeon immediately. Which
contraction decreasing venous return c. Deep vein thrombosis (DVT) electrolyte disturbance most commonly follows thyroid surgery?
due to immobility of the ipsilateral side d. Hypoalbuminemia due to a. Hypocalcemia b. Hyponatremia c. Hyperkalemia
protein escaping from an inflamed glomerulus Nursing Crib – Student Nurses’ Community 62
64. Heberden’s nodes are a common sign of osteoarthritis. Which of d. Hypermagnesemia
the following statement is correct about this deformity? 77. Which laboratory test value is elevated in clients who smoke and
a. It appears only in men b. It appears on the distal interphalangeal can't be used as a general indicator of cancer?
joint c. It appears on the proximal interphalangeal joint d. It appears a. Acid phosphatase level b. Serum calcitonin level c. Alkaline
on the dorsolateral aspect of the interphalangeal joint. phosphatase level d. Carcinoembryonic antigen level
65. Which of the following statements explains the main difference 78. Francis with anemia has been admitted to the medical-surgical
between rheumatoid arthritis and osteoarthritis? unit. Which assessment findings are characteristic of iron-deficiency
a. Osteoarthritis is gender-specific, rheumatoid arthritis isn’t b. anemia?
Osteoarthritis is a localized disease rheumatoid arthritis is systemic a. Nights sweats, weight loss, and diarrhea b. Dyspnea, tachycardia,
c. Osteoarthritis is a systemic disease, rheumatoid arthritis is and pallor c. Nausea, vomiting, and anorexia d. Itching, rash, and
localized d. Osteoarthritis has dislocations and subluxations, jaundice
rheumatoid arthritis doesn’t 79. In teaching a female client who is HIV-positive about pregnancy,
66. Mrs. Cruz uses a cane for assistance in walking. Which of the the nurse would know more teaching is necessary when the client
following statements is true about a cane or other assistive devices? says:
Nursing Crib – Student Nurses’ Community 60 a. The baby can get the virus from my placenta." b. "I'm planning on
a. A walker is a better choice than a cane. b. The cane should be starting on birth control pills." c. "Not everyone who has the virus
used on the affected side c. The cane should be used on the gives birth to a baby who has the virus." d. "I'll need to have a C-
unaffected side d. A client with osteoarthritis should be encouraged section if I become pregnant and have a baby."
to ambulate without the cane 80. When preparing Judy with acquired immunodeficiency syndrome
67. A male client with type 1 diabetes is scheduled to receive 30 U of (AIDS) for discharge to the home, the nurse should be sure to
70/30 insulin. There is no 70/30 insulin available. As a substitution, include which instruction?
the nurse may give the client: a. 9 U regular insulin and 21 U neutral a. "Put on disposable gloves before bathing." b. "Sterilize all plates
protamine Hagedorn (NPH). b. 21 U regular insulin and 9 U NPH. c. and utensils in boiling water." c. "Avoid sharing such articles as
10 U regular insulin and 20 U NPH. d. 20 U regular insulin and 10 U toothbrushes and razors." d. "Avoid eating foods from serving dishes
NPH. shared by other family members."
81. Nurse Marie is caring for a 32-year-old client admitted with 92. Nurse Sarah is caring for clients on the surgical floor and has just
pernicious anemia. Which set of findings should the nurse expect received report from the previous shift. Which of the following
when assessing the client? clients should the nurse see first?
a. Pallor, bradycardia, and reduced pulse pressure b. Pallor, a. A 35-year-old admitted three hours ago with a gunshot wound;
tachycardia, and a sore tongue c. Sore tongue, dyspnea, and weight 1.5 cm area of dark drainage noted on the dressing. b. A 43-year-old
gain d. Angina, double vision, and anorexia who had a mastectomy two days ago; 23 ml of serosanguinous fluid
Nursing Crib – Student Nurses’ Community 63 noted in the Jackson-Pratt drain. c. A 59-year-old with a collapsed
82. After receiving a dose of penicillin, a client develops dyspnea and lung due to an accident; no drainage noted in the previous eight
hypotension. Nurse Celestina suspects the client is experiencing hours. d. A 62-year-old who had an abdominal-perineal resection
anaphylactic shock. What should the nurse do first? three days ago; client complaints of chills.
a. Page an anesthesiologist immediately and prepare to intubate the 93. Nurse Eve is caring for a client who had a thyroidectomy 12
client. b. Administer epinephrine, as prescribed, and prepare to hours ago for treatment of Grave’s disease. The nurse would be
intubate the client if necessary. c. Administer the antidote for most concerned if which of the following was observed?
penicillin, as prescribed, and continue to monitor the client's vital a. Blood pressure 138/82, respirations 16, oral temperature 99
signs. d. Insert an indwelling urinary catheter and begin to infuse I.V. degrees Fahrenheit. b. The client supports his head and neck when
fluids as ordered. turning his head to the right. c. The client spontaneously flexes his
83. Mr. Marquez with rheumatoid arthritis is about to begin aspirin wrist when the blood pressure is obtained. d. The client is drowsy
therapy to reduce inflammation. When teaching the client about and complains of sore throat.
aspirin, the nurse discusses adverse reactions to prolonged aspirin 94. Julius is admitted with complaints of severe pain in the lower
therapy. These include: right quadrant of the abdomen. To assist with pain relief, the nurse
a. weight gain. b. fine motor tremors. c. respiratory acidosis. d. should take which of the following actions?
bilateral hearing loss. a. Encourage the client to change positions frequently in bed. b.
84. A 23-year-old client is diagnosed with human immunodeficiency Administer Demerol 50 mg IM q 4 hours and PRN. c. Apply warmth
virus (HIV). After recovering from the initial shock of the diagnosis, to the abdomen with a heating pad. d. Use comfort measures and
the client expresses a desire to learn as much as possible about HIV pillows to position the client.
and acquired immunodeficiency syndrome (AIDS). When teaching 95. Nurse Tina prepares a client for peritoneal dialysis. Which of the
the client about the immune system, the nurse states that adaptive following actions should the nurse take first?
immunity is provided by which type of white blood cell? a. Assess for a bruit and a thrill. b. Warm the dialysate solution. c.
a. Neutrophil b. Basophil c. Monocyte d. Lymphocyte Position the client on the left side. d. Insert a Foley catheter
85. In an individual with Sjögren's syndrome, nursing care should Nursing Crib – Student Nurses’ Community 66
focus on: 96. Nurse Jannah teaches an elderly client with right-sided weakness
a. moisture replacement. b. electrolyte balance. c. nutritional how to use cane. Which of the following behaviors, if demonstrated
supplementation. d. arrhythmia management. by the client to the nurse, indicates that the teaching was effective?
86. During chemotherapy for lymphocytic leukemia, Mathew a. The client holds the cane with his right hand, moves the can
develops abdominal pain, fever, and "horse barn" smelling diarrhea. forward followed by the right leg, and then moves the left leg. b. The
It would be most important for the nurse to advise the physician to client holds the cane with his right hand, moves the cane forward
order: followed by his left leg, and then moves the right leg. c. The client
a. enzyme-linked immunosuppressant assay (ELISA) test. b. holds the cane with his left hand, moves the cane forward followed
electrolyte panel and hemogram. by the right leg, and then moves the left leg. d. The client holds the
Nursing Crib – Student Nurses’ Community 64 cane with his left hand, moves the cane forward followed by his left
c. stool for Clostridium difficile test. d. flat plate X-ray of the leg, and then moves the right leg.
abdomen. 97. An elderly client is admitted to the nursing home setting. The
87. A male client seeks medical evaluation for fatigue, night sweats, client is occasionally confused and her gait is often unsteady. Which
and a 20-lb weight loss in 6 weeks. To confirm that the client has of the following actions, if taken by the nurse, is most appropriate?
been infected with the human immunodeficiency virus (HIV), the a. Ask the woman’s family to provide personal items such as photos
nurse expects the physician to order: or mementos. b. Select a room with a bed by the door so the woman
a. E-rosette immunofluorescence. b. quantification of T- can look down the hall. c. Suggest the woman eat her meals in the
lymphocytes. c. enzyme-linked immunosorbent assay (ELISA). d. room with her roommate. d. Encourage the woman to ambulate in
Western blot test with ELISA. the halls twice a day.
88. A complete blood count is commonly performed before a Joe 98. Nurse Evangeline teaches an elderly client how to use a standard
goes into surgery. What does this test seek to identify? aluminum walker. Which of the following behaviors, if demonstrated
a. Potential hepatic dysfunction indicated by decreased blood urea by the client, indicates that the nurse’s teaching was effective?
nitrogen (BUN) and creatinine levels b. Low levels of urine a. The client slowly pushes the walker forward 12 inches, then takes
constituents normally excreted in the urine c. Abnormally low small steps forward while leaning on the walker. b. The client lifts
hematocrit (HCT) and hemoglobin (Hb) levels d. Electrolyte the walker, moves it forward 10 inches, and then takes several small
imbalance that could affect the blood's ability to coagulate properly steps forward. c. The client supports his weight on the walker while
89. While monitoring a client for the development of disseminated advancing it forward, then takes small steps while balancing on the
intravascular coagulation (DIC), the nurse should take note of what walker. d. The client slides the walker 18 inches forward, then takes
assessment parameters? small steps while holding onto the walker for balance.
a. Platelet count, prothrombin time, and partial thromboplastin time 99. Nurse Deric is supervising a group of elderly clients in a
b. Platelet count, blood glucose levels, and white blood cell (WBC) residential home setting. The nurse knows that the elderly are at
count c. Thrombin time, calcium levels, and potassium levels d. greater risk of developing sensory deprivation for what reason?
Fibrinogen level, WBC, and platelet count a. Increased sensitivity to the side effects of medications. b.
90. When taking a dietary history from a newly admitted female Decreased visual, auditory, and gustatory abilities. c. Isolation from
client, Nurse Len should remember that which of the following foods their families and familiar surroundings. d. Decrease musculoskeletal
is a common allergen? function and mobility.
a. Bread b. Carrots c. Orange d. Strawberries Nursing Crib – Student Nurses’ Community 67
91. Nurse John is caring for clients in the outpatient clinic. Which of 100. A male client with emphysema becomes restless and confused.
the following phone calls should the nurse return first? What step should nurse Jasmine take next?
a. A client with hepatitis A who states, “My arms and legs are a. Encourage the client to perform pursed lip breathing. b. Check the
itching.” b. A client with cast on the right leg who states, “I have a client’s temperature. c. Assess the client’s potassium level. d.
funny feeling in my right leg.” c. A client with osteomyelitis of the Increase the client’s oxygen flow rate.
spine who states, “I am so nauseous that I can’t eat.”
Nursing Crib – Student Nurses’ Community 65
d. A client with rheumatoid arthritis who states, “I am having trouble
sleeping.”
NURSING PRACTICE IV using a non-rebreathing mask d. Blood gases are monitored using a
Care of Clients with Physiologic and Psychosocial Alterations pulse oximeter.
TEST IV - Care of Clients with Physiologic and Psychosocial 14.Tonny has undergoes a left thoracotomy and a partial
Alterations pneumonectomy. Chest tubes are inserted, and one-bottle water-
1. Randy has undergone kidney transplant, what assessment would seal drainage is instituted in the operating room. In the
prompt Nurse Katrina to suspect organ rejection? postanesthesia care unit Tonny is placed in Fowler's position on
a. Sudden weight loss b. Polyuria c. Hypertension d. Shock either his right side or on his back. The nurse is aware that this
2. The immediate objective of nursing care for an overweight, mildly position:
hypertensive male client with ureteral colic and hematuria is to a. Reduce incisional pain. b. Facilitate ventilation of the left lung. c.
decrease: Equalize pressure in the pleural space. d. Increase venous return
a. Pain b. Weight c. Hematuria d. Hypertension 15.Kristine is scheduled for a bronchoscopy. When teaching Kristine
3. Matilda, with hyperthyroidism is to receive Lugol’s iodine solution what to expect afterward, the nurse's highest priority of information
before a subtotal thyroidectomy is performed. The nurse is aware would be:
that this medication is given to: Nursing Crib – Student Nurses’ Community 72
a. Decrease the total basal metabolic rate. b. Maintain the function a. Food and fluids will be withheld for at least 2 hours. b. Warm
of the parathyroid glands. c. Block the formation of thyroxine by the saline gargles will be done q 2h. c. Coughing and deep-breathing
thyroid gland. d. Decrease the size and vascularity of the thyroid exercises will be done q2h. d. Only ice chips and cold liquids will be
gland. allowed initially.
4. Ricardo, was diagnosed with type I diabetes. The nurse is aware 16.Nurse Tristan is caring for a male client in acute renal failure. The
that acute hypoglycemia also can develop in the client who is nurse should expect hypertonic glucose, insulin infusions, and
diagnosed with: sodium bicarbonate to be used to treat:
a. Liver disease b. Hypertension c. Type 2 diabetes d. a. hypernatremia. b. hypokalemia. c. hyperkalemia. d.
Hyperthyroidism hypercalcemia.
5. Tracy is receiving combination chemotherapy for treatment of 17.Ms. X has just been diagnosed with condylomata acuminata
metastatic carcinoma. Nurse Ruby should monitor the client for the (genital warts). What information is appropriate to tell this client?
systemic side effect of: a. This condition puts her at a higher risk for cervical cancer;
a. Ascites b. Nystagmus c. Leukopenia d. Polycythemia therefore, she should have a Papanicolaou (Pap) smear annually. b.
Nursing Crib – Student Nurses’ Community 70 The most common treatment is metronidazole (Flagyl), which should
6. Norma, with recent colostomy expresses concern about the eradicate the problem within 7 to 10 days. c. The potential for
inability to control the passage of gas. Nurse Oliver should suggest transmission to her sexual partner will be eliminated if condoms are
that the client plan to: used every time they have sexual intercourse. d. The human
a. Eliminate foods high in cellulose. b. Decrease fluid intake at meal papillomavirus (HPV), which causes condylomata acuminata, can't
times. c. Avoid foods that in the past caused flatus. d. Adhere to a be transmitted during oral sex.
bland diet prior to social events. 18.Maritess was recently diagnosed with a genitourinary problem
7. Nurse Ron begins to teach a male client how to perform and is being examined in the emergency department. When
colostomy irrigations. The nurse would evaluate that the instructions palpating the her kidneys, the nurse should keep which anatomical
were understood when the client states, “I should: fact in mind?
a. Lie on my left side while instilling the irrigating solution.” b. Keep a. The left kidney usually is slightly higher than the right one. b. The
the irrigating container less than 18 inches above the stoma.” c. kidneys are situated just above the adrenal glands. c. The average
Instill a minimum of 1200 ml of irrigating solution to stimulate kidney is approximately 5 cm (2") long and 2 to 3 cm (¾" to 1-1/8")
evacuation of the bowel.” d. Insert the irrigating catheter deeper wide. d. The kidneys lie between the 10th and 12th thoracic
into the stoma if cramping occurs during the procedure.” vertebrae.
8. Patrick is in the oliguric phase of acute tubular necrosis and is 19.Jestoni with chronic renal failure (CRF) is admitted to the urology
experiencing fluid and electrolyte imbalances. The client is unit. The nurse is aware that the diagnostic test are consistent with
somewhat confused and complains of nausea and muscle weakness. CRF if the result is:
As part of the prescribed therapy to correct this electrolyte a. Increased pH with decreased hydrogen ions. b. Increased serum
imbalance, the nurse would expect to: levels of potassium, magnesium, and calcium. c. Blood urea nitrogen
a. Administer Kayexalate b. Restrict foods high in protein c. Increase (BUN) 100 mg/dl and serum creatinine 6.5 mg/ dl.
oral intake of cheese and milk. d. Administer large amounts of Nursing Crib – Student Nurses’ Community 73
normal saline via I.V. d. Uric acid analysis 3.5 mg/dl and phenolsulfonphthalein (PSP)
9. Mario has burn injury. After Forty48 hours, the physician orders excretion 75%. 20.Katrina has an abnormal result on a Papanicolaou
for Mario 2 liters of IV fluid to be administered q12 h. The drop test. After admitting that she read her chart while the nurse was out
factor of the tubing is 10 gtt/ml. The nurse should set the flow to of the room, Katrina asks what dysplasia means. Which definition
provide: should the nurse provide?
a. 18 gtt/min b. 28 gtt/min c. 32 gtt/min d. 36 gtt/min a. Presence of completely undifferentiated tumor cells that don't
10.Terence suffered form burn injury. Using the rule of nines, which resemble cells of the tissues of their origin. b. Increase in the
has the largest percent of burns? number of normal cells in a normal arrangement in a tissue or an
a. Face and neck b. Right upper arm and penis organ. c. Replacement of one type of fully differentiated cell by
Nursing Crib – Student Nurses’ Community 71 another in tissues where the second type normally isn't found. d.
c. Right thigh and penis d. Upper trunk Alteration in the size, shape, and organization of differentiated cells.
11.Herbert, a 45 year old construction engineer is brought to the 21.During a routine checkup, Nurse Mariane assesses a male client
hospital unconscious after falling from a 2-story building. When with acquired immunodeficiency syndrome (AIDS) for signs and
assessing the client, the nurse would be most concerned if the symptoms of cancer. What is the most common AIDS-related
assessment revealed: cancer?
a. Reactive pupils b. A depressed fontanel c. Bleeding from a. Squamous cell carcinoma b. Multiple myeloma c. Leukemia d.
ears d. An elevated temperature Kaposi's sarcoma
12.Nurse Sherry is teaching male client regarding his permanent 22.Ricardo is scheduled for a prostatectomy, and the
artificial pacemaker. Which information given by the nurse shows anesthesiologist plans to use a spinal (subarachnoid) block during
her knowledge deficit about the artificial cardiac pacemaker? surgery. In the operating room, the nurse positions the client
a. take the pulse rate once a day, in the morning upon awakening according to the anesthesiologist's instructions. Why does the client
b. May be allowed to use electrical appliances c. Have regular require special positioning for this type of anesthesia?
follow up care d. May engage in contact sports a. To prevent confusion b. To prevent seizures c. To prevent
13.The nurse is ware that the most relevant knowledge about cerebrospinal fluid (CSF) leakage d. To prevent cardiac arrhythmias
oxygen administration to a male client with COPD is 23.A male client had a nephrectomy 2 days ago and is now
a. Oxygen at 1-2L/min is given to maintain the hypoxic stimulus for complaining of abdominal pressure and nausea. The first nursing
breathing. b. Hypoxia stimulates the central chemoreceptors in the action should be to:
medulla that makes the client breath. c. Oxygen is administered best a. Auscultate bowel sounds. b. Palpate the abdomen. c. Change the
client's position. d. Insert a rectal tube.
24.Wilfredo with a recent history of rectal bleeding is being beats/minute with a palpable pulse. Which of the following actions
prepared for a colonoscopy. How should the nurse Patricia position should the nurse take first?
the client for this test initially? a. Start an L.V. line and administer amiodarone (Cardarone), 300 mg
Nursing Crib – Student Nurses’ Community 74 L.V. over 10 minutes. b. Check endotracheal tube placement. c.
a. Lying on the right side with legs straight b. Lying on the left side Obtain an arterial blood gas (ABG) sample. d. Administer atropine, 1
with knees bent c. Prone with the torso elevated d. Bent over with mg L.V.
hands touching the floor 36. After cardiac surgery, a client’s blood pressure measures 126/80
25.A male client with inflammatory bowel disease undergoes an mm Hg. Nurse Katrina determines that mean arterial pressure (MAP)
ileostomy. On the first day after surgery, Nurse Oliver notes that the is which of the following?
client's stoma appears dusky. How should the nurse interpret this a. 46 mm Hg b. 80 mm Hg c. 95 mm Hg d. 90 mm Hg
finding? 37. A female client arrives at the emergency department with chest
a. Blood supply to the stoma has been interrupted. b. This is a and stomach pain and a report of black tarry stool for several
normal finding 1 day after surgery. c. The ostomy bag should be months. Which of the following order should the nurse Oliver
adjusted. d. An intestinal obstruction has occurred. anticipate?
26.Anthony suffers burns on the legs, which nursing intervention a. Cardiac monitor, oxygen, creatine kinase and lactate
helps prevent contractures? dehydrogenase levels b. Prothrombin time, partial thromboplastin
a. Applying knee splints b. Elevating the foot of the bed c. time, fibrinogen and fibrin split product values.
Hyperextending the client's palms d. Performing shoulder range-of- Nursing Crib – Student Nurses’ Community 77
motion exercises c. Electrocardiogram, complete blood count, testing for occult blood,
27.Nurse Ron is assessing a client admitted with second- and third- comprehensive serum metabolic panel. d. Electroencephalogram,
degree burns on the face, arms, and chest. Which finding indicates a alkaline phosphatase and aspartate aminotransferase levels, basic
potential problem? serum metabolic panel
a. Partial pressure of arterial oxygen (PaO2) value of 80 mm Hg. b. 38. Macario had coronary artery bypass graft (CABG) surgery 3 days
Urine output of 20 ml/hour. c. White pulmonary secretions. d. Rectal ago. Which of the following conditions is suspected by the nurse
temperature of 100.6° F (38° C). when a decrease in platelet count from 230,000 ul to 5,000 ul is
28.Mr. Mendoza who has suffered a cerebrovascular accident (CVA) noted?
is too weak to move on his own. To help the client avoid pressure a. Pancytopenia b. Idiopathic thrombocytopemic purpura (ITP) c.
ulcers, Nurse Celia should: Disseminated intravascular coagulation (DIC) d. Heparin-associated
a. Turn him frequently. b. Perform passive range-of-motion (ROM) thrombosis and thrombocytopenia (HATT)
exercises. c. Reduce the client's fluid intake. d. Encourage the client 39. Which of the following drugs would be ordered by the physician
to use a footboard. to improve the platelet count in a male client with idiopathic
Nursing Crib – Student Nurses’ Community 75 thrombocytopenic purpura (ITP)?
29.Nurse Maria plans to administer dexamethasone cream to a a. Acetylsalicylic acid (ASA) b. Corticosteroids c. Methotrezate d.
female client who has dermatitis over the anterior chest. How Vitamin K
should the nurse apply this topical agent? 40. A female client is scheduled to receive a heart valve replacement
a. With a circular motion, to enhance absorption. b. With an upward with a porcine valve. Which of the following types of transplant is
motion, to increase blood supply to the affected area c. In long, this?
even, outward, and downward strokes in the direction of hair a. Allogeneic b. Autologous c. Syngeneic d. Xenogeneic
growth d. In long, even, outward, and upward strokes in the 41. Marco falls off his bicycle and injuries his ankle. Which of the
direction opposite hair growth following actions shows the initial response to the injury in the
30.Nurse Kate is aware that one of the following classes of extrinsic pathway?
medication protect the ischemic myocardium by blocking a. Release of Calcium b. Release of tissue thromboplastin c.
catecholamines and sympathetic nerve stimulation is: Conversion of factors XII to factor XIIa d. Conversion of factor VIII to
a. Beta -adrenergic blockers b. Calcium channel blocker c. Narcotics factor VIIIa
d. Nitrates 42. Instructions for a client with systemic lupus erythematosus (SLE)
31.A male client has jugular distention. On what position should the would include information about which of the following blood
nurse place the head of the bed to obtain the most accurate reading dyscrasias?
of jugular vein distention? a. Dressler’s syndrome b. Polycythemia c. Essential
a. High Fowler’s b. Raised 10 degrees c. Raised 30 degrees d. Supine thrombocytopenia
position Nursing Crib – Student Nurses’ Community 78
32.The nurse is aware that one of the following classes of d. Von Willebrand’s disease
medications maximizes cardiac performance in clients with heart 43. The nurse is aware that the following symptoms is most
failure by increasing ventricular contractility? commonly an early indication of stage 1 Hodgkin’s disease?
a. Beta-adrenergic blockers b. Calcium channel blocker c. Diuretics d. a. Pericarditis b. Night sweat c. Splenomegaly d. Persistent
Inotropic agents hypothermia
33.A male client has a reduced serum high-density lipoprotein (HDL) 44. Francis with leukemia has neutropenia. Which of the following
level and an elevated low-density lipoprotein (LDL) level. Which of functions must frequently assessed?
the following dietary modifications is not appropriate for this client? a. Blood pressure b. Bowel sounds c. Heart sounds d. Breath sounds
a. Fiber intake of 25 to 30 g daily 45. The nurse knows that neurologic complications of multiple
Nursing Crib – Student Nurses’ Community 76 myeloma (MM) usually involve which of the following body system?
b. Less than 30% of calories form fat c. Cholesterol intake of less a. Brain b. Muscle spasm c. Renal dysfunction d. Myocardial
than 300 mg daily d. Less than 10% of calories from saturated fat 34. irritability
A 37-year-old male client was admitted to the coronary care unit 46. Nurse Patricia is aware that the average length of time from
(CCU) 2 days ago with an acute myocardial infarction. Which of the human immunodeficiency virus (HIV) infection to the development
following actions would breach the client confidentiality? of acquired immunodeficiency syndrome (AIDS)?
a. The CCU nurse gives a verbal report to the nurse on the telemetry a. Less than 5 years b. 5 to 7 years c. 10 years d. More than 10 years
unit before transferring the client to that unit b. The CCU nurse 47. An 18-year-old male client admitted with heat stroke begins to
notifies the on-call physician about a change in the client’s condition show signs of disseminated intravascular coagulation (DIC). Which of
c. The emergency department nurse calls up the latest the following laboratory findings is most consistent with DIC?
electrocardiogram results to check the client’s progress. d. At the a. Low platelet count b. Elevated fibrinogen levels c. Low levels of
client’s request, the CCU nurse updates the client’s wife on his fibrin degradation products d. Reduced prothrombin time
condition Nursing Crib – Student Nurses’ Community 79
35. A male client arriving in the emergency department is receiving 48. Mario comes to the clinic complaining of fever, drenching night
cardiopulmonary resuscitation from paramedics who are giving sweats, and unexplained weight loss over the past 3 months.
ventilations through an endotracheal (ET) tube that they placed in Physical examination reveals a single enlarged supraclavicular lymph
the client’s home. During a pause in compressions, the cardiac node. Which of the following is the most probable diagnosis?
monitor shows narrow QRS complexes and a heart rate of a. Influenza b. Sickle cell anemia c. Leukemia d. Hodgkin’s disease
49. A male client with a gunshot wound requires an emergency a. “I’ll see if your physician is in the hospital”. b. “Maybe your
blood transfusion. His blood type is AB negative. Which blood type reacting to the drug; I will withhold the next dose”.
would be the safest for him to receive? Nursing Crib – Student Nurses’ Community 82
a. AB Rh-positive b. A Rh-positive c. A Rh-negative d. O Rh-positive c. “I’ll lower the dosage as ordered so the drug causes only 2 to 4
Situation: Stacy is diagnosed with acute lymphoid leukemia (ALL) and stools a day”. d. “Frequently, bowel movements are needed to
beginning chemotherapy. 50. Stacy is discharged from the hospital reduce sodium level”.
following her chemotherapy treatments. Which statement of Stacy’s 63. Which of the following groups of symptoms indicates a ruptured
mother indicated that she understands when she will contact the abdominal aortic aneurysm?
physician? a. Lower back pain, increased blood pressure, decreased re blood
a. “I should contact the physician if Stacy has difficulty in sleeping”. cell (RBC) count, increased white blood (WBC) count. b. Severe lower
b. “I will call my doctor if Stacy has persistent vomiting and back pain, decreased blood pressure, decreased RBC count,
diarrhea”. c. “My physician should be called if Stacy is irritable and increased WBC count. c. Severe lower back pain, decreased blood
unhappy”. d. “Should Stacy have continued hair loss, I need to call pressure, decreased RBC count, decreased RBC count, decreased
the doctor”. WBC count. d. Intermitted lower back pain, decreased blood
51. Stacy’s mother states to the nurse that it is hard to see Stacy pressure, decreased RBC count, increased WBC count.
with no hair. The best response for the nurse is: 64. After undergoing a cardiac catheterization, Tracy has a large
a. “Stacy looks very nice wearing a hat”. b. “You should not worry puddle of blood under his buttocks. Which of the following steps
about her hair, just be glad that she is alive”. c. “Yes it is upsetting. should the nurse take first?
But try to cover up your feelings when you are with her or else she a. Call for help. b. Obtain vital signs c. Ask the client to “lift up” d.
may be upset”. d. “This is only temporary; Stacy will re-grow new Apply gloves and assess the groin site
hair in 3-6 months, but may be different in texture”. 65. Which of the following treatment is a suitable surgical
52. Stacy has beginning stomatitis. To promote oral hygiene and intervention for a client with unstable angina?
comfort, the nurse in-charge should: a. Cardiac catheterization b. Echocardiogram c. Nitroglycerin d.
a. Provide frequent mouthwash with normal saline. Percutaneous transluminal coronary angioplasty (PTCA)
Nursing Crib – Student Nurses’ Community 80 66. The nurse is aware that the following terms used to describe
b. Apply viscous Lidocaine to oral ulcers as needed. c. Use lemon reduced cardiac output and perfusion impairment due to ineffective
glycerine swabs every 2 hours. d. Rinse mouth with Hydrogen pumping of the heart is:
Peroxide. a. Anaphylactic shock b. Cardiogenic shock c. Distributive shock d.
53. During the administration of chemotherapy agents, Nurse Oliver Myocardial infarction (MI)
observed that the IV site is red and swollen, when the IV is touched 67. A client with hypertension ask the nurse which factors can cause
Stacy shouts in pain. The first nursing action to take is: blood pressure to drop to normal levels?
a. Notify the physician b. Flush the IV line with saline solution c. a. Kidneys’ excretion to sodium only. b. Kidneys’ retention of sodium
Immediately discontinue the infusion d. Apply an ice pack to the site, and water c. Kidneys’ excretion of sodium and water
followed by warm compress. Nursing Crib – Student Nurses’ Community 83
54. The term “blue bloater” refers to a male client which of the d. Kidneys’ retention of sodium and excretion of water
following conditions? 68. Nurse Rose is aware that the statement that best explains why
a. Adult respiratory distress syndrome (ARDS) b. Asthma c. Chronic furosemide (Lasix) is administered to treat hypertension is:
obstructive bronchitis d. Emphysema a. It dilates peripheral blood vessels. b. It decreases sympathetic
55. The term “pink puffer” refers to the female client with which of cardioacceleration. c. It inhibits the angiotensin-coverting enzymes
the following conditions? d. It inhibits reabsorption of sodium and water in the loop of Henle.
a. Adult respiratory distress syndrome (ARDS) b. Asthma c. Chronic 69. Nurse Nikki knows that laboratory results supports the diagnosis
obstructive bronchitis d. Emphysema of systemic lupus erythematosus (SLE) is:
56. Jose is in danger of respiratory arrest following the a. Elavated serum complement level b. Thrombocytosis, elevated
administration of a narcotic analgesic. An arterial blood gas value is sedimentation rate c. Pancytopenia, elevated antinuclear antibody
obtained. Nurse Oliver would expect the paco2 to be which of the (ANA) titer d. Leukocysis, elevated blood urea nitrogen (BUN) and
following values? creatinine levels
a. 15 mm Hg b. 30 mm Hg c. 40 mm Hg d. 80 mm Hg 70. Arnold, a 19-year-old client with a mild concussion is discharged
57. Timothy’s arterial blood gas (ABG) results are as follows; pH from the emergency department. Before discharge, he complains of
7.16; Paco2 80 mm Hg; Pao2 46 mm Hg; HCO3- 24mEq/L; Sao2 81%. a headache. When offered acetaminophen, his mother tells the
This ABG result represents which of the following conditions? nurse the headache is severe and she would like her son to have
a. Metabolic acidosis b. Metabolic alkalosis c. Respiratory acidosis something stronger. Which of the following responses by the nurse
Nursing Crib – Student Nurses’ Community 81 is appropriate?
d. Respirator y alkalosis a. “Your son had a mild concussion, acetaminophen is strong
58. Norma has started a new drug for hypertension. Thirty minutes enough.” b. “Aspirin is avoided because of the danger of Reye’s
after she takes the drug, she develops chest tightness and becomes syndrome in children or young adults.” c. “Narcotics are avoided
short of breath and tachypneic. She has a decreased level of after a head injury because they may hide a worsening condition.” d.
consciousness. These signs indicate which of the following Stronger medications may lead to vomiting, which increases the
conditions? intracarnial pressure (ICP).”
a. Asthma attack b. Pulmonary embolism c. Respiratory failure d. 71. When evaluating an arterial blood gas from a male client with a
Rheumatoid arthritis subdural hematoma, the nurse notes the Paco2 is 30 mm Hg. Which
Situation: Mr. Gonzales was admitted to the hospital with ascites of the following responses best describes the result?
and jaundice. To rule out cirrhosis of the liver: 59. Which laboratory a. Appropriate; lowering carbon dioxide (CO2) reduces intracranial
test indicates liver cirrhosis? pressure (ICP) b. Emergent; the client is poorly oxygenated c. Normal
a. Decreased red blood cell count b. Decreased serum acid d. Significant; the client has alveolar hypoventilation
phosphate level c. Elevated white blood cell count d. Elevated serum 72. When prioritizing care, which of the following clients should the
aminotransferase nurse Olivia assess first?
60.The biopsy of Mr. Gonzales confirms the diagnosis of cirrhosis. Nursing Crib – Student Nurses’ Community 84
Mr. Gonzales is at increased risk for excessive bleeding primarily a. A 17-year-old clients 24-hours postappendectomy b. A 33-year-old
because of: client with a recent diagnosis of Guillain-Barre syndrome c. A 50-
a. Impaired clotting mechanism b. Varix formation c. Inadequate year-old client 3 days postmyocardial infarction d. A 50-year-old
nutrition d. Trauma of invasive procedure client with diverticulitis
61. Mr. Gonzales develops hepatic encephalopathy. Which clinical 73. JP has been diagnosed with gout and wants to know why
manifestation is most common with this condition? colchicine is used in the treatment of gout. Which of the following
a. Increased urine output b. Altered level of consciousness c. actions of colchicines explains why it’s effective for gout?
Decreased tendon reflex d. Hypotension a. Replaces estrogen b. Decreases infection c. Decreases
62. When Mr. Gonzales regained consciousness, the physician orders inflammation d. Decreases bone demineralization
50 ml of Lactose p.o. every 2 hours. Mr. Gozales develops diarrhea. 74. Norma asks for information about osteoarthritis. Which of the
The nurse best action would be: following statements about osteoarthritis is correct?
a. Osteoarthritis is rarely debilitating b. Osteoarthritis is a rare form a. No increase in the thyroid-stimulating hormone (TSH) level after
of arthritis c. Osteoarthritis is the most common form of arthritis d. 30 minutes during the TSH stimulation test b. A decreased TSH level
Osteoarthritis afflicts people over 60 c. An increase in the TSH level after 30 minutes during the TSH
75. Ruby is receiving thyroid replacement therapy develops the flu stimulation test d. Below-normal levels of serum triiodothyronine
and forgets to take her thyroid replacement medicine. The nurse (T3) and serum thyroxine (T4) as detected by radioimmunoassay
understands that skipping this medication will put the client at risk 85. Rico with diabetes mellitus must learn how to self-administer
for developing which of the following lifethreatening complications? insulin. The physician has prescribed 10 U of U-100 regular insulin
a. Exophthalmos b. Thyroid storm c. Myxedema coma d. Tibial and 35 U of U-100 isophane insulin suspension (NPH) to be taken
myxedema before breakfast. When teaching the client how to select and rotate
76. Nurse Sugar is assessing a client with Cushing's syndrome. Which insulin injection sites, the nurse should provide which instruction?
observation should the nurse report to the physician immediately? a. "Inject insulin into healthy tissue with large blood vessels and
a. Pitting edema of the legs b. An irregular apical pulse c. Dry nerves." b. "Rotate injection sites within the same anatomic region,
mucous membranes d. Frequent urination not among different regions."
77. Cyrill with severe head trauma sustained in a car accident is Nursing Crib – Student Nurses’ Community 87
admitted to the intensive care unit. Thirty-six hours later, the client's c. "Administer insulin into areas of scar tissue or hypotrophy
urine output suddenly rises above 200 ml/hour, leading the nurse to whenever possible." d. "Administer insulin into sites above muscles
suspect diabetes insipidus. Which laboratory findings support the that you plan to exercise heavily later that day."
nurse's suspicion of diabetes insipidus? 86. Nurse Sarah expects to note an elevated serum glucose level in a
Nursing Crib – Student Nurses’ Community 85 client with hyperosmolar hyperglycemic nonketotic syndrome
a. Above-normal urine and serum osmolality levels b. Below-normal (HHNS). Which other laboratory finding should the nurse anticipate?
urine and serum osmolality levels c. Above-normal urine osmolality a. Elevated serum acetone level b. Serum ketone bodies c. Serum
level, below-normal serum osmolality level d. Below-normal urine alkalosis d. Below-normal serum potassium level
osmolality level, above-normal serum osmolality level 87. For a client with Graves' disease, which nursing intervention
78. Jomari is diagnosed with hyperosmolar hyperglycemic promotes comfort?
nonketotic syndrome (HHNS) is stabilized and prepared for a. Restricting intake of oral fluids b. Placing extra blankets on the
discharge. When preparing the client for discharge and home client's bed c. Limiting intake of high-carbohydrate foods d.
management, which of the following statements indicates that the Maintaining room temperature in the low-normal range
client understands her condition and how to control it? 88. Patrick is treated in the emergency department for a Colles'
a. "I can avoid getting sick by not becoming dehydrated and by fracture sustained during a fall. What is a Colles' fracture?
paying attention to my need to urinate, drink, or eat more than a. Fracture of the distal radius b. Fracture of the olecranon c.
usual." b. "If I experience trembling, weakness, and headache, I Fracture of the humerus d. Fracture of the carpal scaphoid
should drink a glass of soda that contains sugar." c. "I will have to 89. Cleo is diagnosed with osteoporosis. Which electrolytes are
monitor my blood glucose level closely and notify the physician if it's involved in the development of this disorder?
constantly elevated." d. "If I begin to feel especially hungry and a. Calcium and sodium b. Calcium and phosphorous c. Phosphorous
thirsty, I'll eat a snack high in carbohydrates." and potassium d. Potassium and sodium
79. A 66-year-old client has been complaining of sleeping more, 90. Johnny a firefighter was involved in extinguishing a house fire
increased urination, anorexia, weakness, irritability, depression, and and is being treated to smoke inhalation. He develops severe
bone pain that interferes with her going outdoors. Based on these hypoxia 48 hours after the incident, requiring intubation and
assessment findings, the nurse would suspect which of the following mechanical ventilation. He most likely has developed which of the
disorders? following conditions?
a. Diabetes mellitus b. Diabetes insipidus c. Hypoparathyroidism d. a. Adult respiratory distress syndrome (ARDS) b. Atelectasis c.
Hyperparathyroidism Bronchitis
80. Nurse Lourdes is teaching a client recovering from addisonian Nursing Crib – Student Nurses’ Community 88
crisis about the need to take fludrocortisone acetate and d. Pneumonia
hydrocortisone at home. Which statement by the client indicates an 91. A 67-year-old client develops acute shortness of breath and
understanding of the instructions? progressive hypoxia requiring right femur. The hypoxia was probably
a. "I'll take my hydrocortisone in the late afternoon, before dinner." caused by which of the following conditions?
b. "I'll take all of my hydrocortisone in the morning, right after I a. Asthma attack b. Atelectasis c. Bronchitis d. Fat embolism
wake up." c. "I'll take two-thirds of the dose when I wake up and 92. A client with shortness of breath has decreased to absent breath
one-third in the late afternoon." d. "I'll take the entire dose at sounds on the right side, from the apex to the base. Which of the
bedtime." 81..Which of the following laboratory test results would following conditions would best explain this?
suggest to the nurse Len that a client has a corticotropin-secreting a. Acute asthma b. Chronic bronchitis c. Pneumonia d. Spontaneous
pituitary adenoma? pneumothorax
a. High corticotropin and low cortisol levels 93. A 62-year-old male client was in a motor vehicle accident as an
Nursing Crib – Student Nurses’ Community 86 unrestrained driver. He’s now in the emergency department
b. Low corticotropin and high cortisol levels c. High corticotropin and complaining of difficulty of breathing and chest pain. On
high cortisol levels d. Low corticotropin and low cortisol levels auscultation of his lung field, no breath sounds are present in the
82. A male client is scheduled for a transsphenoidal upper lobe. This client may have which of the following conditions?
hypophysectomy to remove a pituitary tumor. Preoperatively, the a. Bronchitis b. Pneumonia c. Pneumothorax d. Tuberculosis (TB)
nurse should assess for potential complications by doing which of 94. If a client requires a pneumonectomy, what fills the area of the
the following? thoracic cavity?
a. Testing for ketones in the urine b. Testing urine specific gravity c. a. The space remains filled with air only b. The surgeon fills the space
Checking temperature every 4 hours d. Performing capillary glucose with a gel c. Serous fluids fills the space and consolidates the region
testing every 4 hours d. The tissue from the other lung grows over to the other side
83. Capillary glucose monitoring is being performed every 4 hours 95. Hemoptysis may be present in the client with a pulmonary
for a client diagnosed with diabetic ketoacidosis. Insulin is embolism because of which of the following reasons?
administered using a scale of regular insulin according to glucose a. Alveolar damage in the infracted area b. Involvement of major
results. At 2 p.m., the client has a capillary glucose level of 250 mg/dl blood vessels in the occluded area c. Loss of lung parenchyma
for which he receives 8 U of regular insulin. Nurse Mariner should Nursing Crib – Student Nurses’ Community 89
expect the dose's: d. Loss of lung tissue
a. onset to be at 2 p.m. and its peak to be at 3 p.m. b. onset to be at 96. Aldo with a massive pulmonary embolism will have an arterial
2:15 p.m. and its peak to be at 3 p.m. c. onset to be at 2:30 p.m. and blood gas analysis performed to determine the extent of hypoxia.
its peak to be at 4 p.m. d. onset to be at 4 p.m. and its peak to be at The acid-base disorder that may be present is?
6 p.m. a. Metabolic acidosis b. Metabolic alkalosis c. Respiratory acidosis d.
84. The physician orders laboratory tests to confirm hyperthyroidism Respiratory alkalosis
in a female client with classic signs and symptoms of this disorder. 97. After a motor vehicle accident, Armand an 22-year-old client is
Which test result would confirm the diagnosis? admitted with a pneumothorax. The surgeon inserts a chest tube
and attaches it to a chest drainage system. Bubbling soon appears in
the water seal chamber. Which of the following is the most likely 8. In preparing a female client for electroconvulsive therapy (ECT),
cause of the bubbling? Nurse Michelle knows that succinylcoline (Anectine) will be
a. Air leak b. Adequate suction c. Inadequate suction d. Kinked chest administered for which therapeutic effect?
tube a. Short-acting anesthesia b. Decreased oral and respiratory
98. Nurse Michelle calculates the IV flow rate for a postoperative secretions. c. Skeletal muscle paralysis. d. Analgesia.
client. The client receives 3,000 ml of Ringer’s lactate solution IV to 9. Nurse Gina is aware that the dietary implications for a client in
run over 24 hours. The IV infusion set has a drop factor of 10 drops manic phase of bipolar disorder is:
per milliliter. The nurse should regulate the client’s IV to deliver how a. Serve the client a bowl of soup, buttered French bread, and apple
many drops per minute? slices. b. Increase calories, decrease fat, and decrease protein. c.
a. 18 b. 21 c. 35 d. 40 Give the client pieces of cut-up steak, carrots, and an apple. d.
99. Mickey, a 6-year-old child with a congenital heart disorder is Increase calories, carbohydrates, and protein.
admitted with congestive heart failure. Digoxin (lanoxin) 0.12 mg is 10.What parental behavior toward a child during an admission
ordered for the child. The bottle of Lanoxin contains .05 mg of procedure should cause Nurse Ron to suspect child abuse?
Lanoxin in 1 ml of solution. What amount should the nurse Nursing Crib – Student Nurses’ Community 94
administer to the child? a. Flat affect b. Expressing guilt c. Acting overly solicitous toward the
a. 1.2 ml b. 2.4 ml c. 3.5 ml d. 4.2 ml child. d. Ignoring the child.
100. Nurse Alexandra teaches a client about elastic stockings. Which 11.Nurse Lynnette notices that a female client with obsessive-
of the following statements, if made by the client, indicates to the compulsive disorder washes her hands for long periods each day.
nurse that the teaching was successful? How should the nurse respond to this compulsive behavior?
Nursing Crib – Student Nurses’ Community 90 a. By designating times during which the client can focus on the
a. “I will wear the stockings until the physician tells me to remove behavior. b. By urging the client to reduce the frequency of the
them.” b. “I should wear the stockings even when I am sleep.” c. behavior as rapidly as possible. c. By calling attention to or
“Every four hours I should remove the stockings for a half hour.” d. attempting to prevent the behavior. d. By discouraging the client
“I should put on the stockings before getting out of bed in the from verbalizing anxieties.
morning.” 12.After seeking help at an outpatient mental health clinic, Ruby
Nursing Crib – Student Nurses’ Community 91 who was raped while walking her dog is diagnosed with
posttraumatic stress disorder (PTSD). Three months later, Ruby
NURSING PRACTICE V returns to the clinic, complaining of fear, loss of control, and helpless
Care of Clients with Physiologic and Psychosocial Alterations feelings. Which nursing intervention is most appropriate for Ruby?
Nursing Crib – Student Nurses’ Community 92 a. Recommending a high-protein, low-fat diet. b. Giving sleep
TEST V - Care of Clients with Physiologic and Psychosocial Alterations medication, as prescribed, to restore a normal sleepwake cycle. c.
Allowing the client time to heal. d. Exploring the meaning of the
1. Mr. Marquez reports of losing his job, not being able to sleep at traumatic event with the client.
night, and feeling upset with his wife. Nurse John responds to the 13.Meryl, age 19, is highly dependent on her parents and fears
client, “You may want to talk about your employment situation in leaving home to go away to college. Shortly before the semester
group today.” The Nurse is using which therapeutic technique? starts, she complains that her legs are paralyzed and is rushed to the
a. Observations b. Restating c. Exploring d. Focusing emergency department. When physical examination rules out a
2. Tony refuses his evening dose of Haloperidol (Haldol), then physical cause for her paralysis, the physician admits her to the
becomes extremely agitated in the dayroom while other clients are psychiatric unit where she is diagnosed with conversion disorder.
watching television. He begins cursing and throwing furniture. Nurse Meryl asks the nurse, "Why has this happened to me?" What is the
Oliver first action is to: nurse's best response?
a. Check the client’s medical record for an order for an as-needed a. "You've developed this paralysis so you can stay with your
I.M. dose of medication for agitation. b. Place the client in full parents. You must deal with this conflict if you want to walk again."
leather restraints. c. Call the attending physician and report the b. "It must be awful not to be able to move your legs. You may feel
behavior. d. Remove all other clients from the dayroom. better if you realize the problem is psychological, not physical." c.
3. Tina who is manic, but not yet on medication, comes to the drug "Your problem is real but there is no physical basis for it. We'll work
treatment center. The nurse would not let this client join the group on what is going on in your life to find out why it's happened." d. "It
session because: isn't uncommon for someone with your personality to develop a
a. The client is disruptive. b. The client is harmful to self. c. The client conversion disorder during times of stress."
is harmful to others. d. The client needs to be on medication first. Nursing Crib – Student Nurses’ Community 95
4. Dervid, an adolescent boy was admitted for substance abuse and 14.Nurse Krina knows that the following drugs have been known to
hallucinations. The client’s mother asks Nurse Armando to talk with be effective in treating obsessive-compulsive disorder (OCD):
his husband when he arrives at the hospital. The mother says that a. benztropine (Cogentin) and diphenhydramine (Benadryl). b.
she is afraid of what the father might say to the boy. The most chlordiazepoxide (Librium) and diazepam (Valium) c. fluvoxamine
appropriate nursing intervention would be to: (Luvox) and clomipramine (Anafranil) d. divalproex (Depakote) and
a. Inform the mother that she and the father can work through this lithium (Lithobid)
problem themselves. b. Refer the mother to the hospital social 15.Alfred was newly diagnosed with anxiety disorder. The physician
worker. c. Agree to talk with the mother and the father together. d. prescribed buspirone (BuSpar). The nurse is aware that the teaching
Suggest that the father and son work things out. instructions for newly prescribed buspirone should include which of
5. What is Nurse John likely to note in a male client being admitted the following?
for alcohol withdrawal? a. A warning about the drugs delayed therapeutic effect, which is
Nursing Crib – Student Nurses’ Community 93 from 14 to 30 days. b. A warning about the incidence of neuroleptic
a. Perceptual disorders. b. Impending coma. c. Recent alcohol intake. malignant syndrome (NMS). c. A reminder of the need to schedule
d. Depression with mutism. blood work in 1 week to check blood levels of the drug. d. A warning
6. Aira has taken amitriptyline HCL (Elavil) for 3 days, but now that immediate sedation can occur with a resultant drop in pulse.
complains that it “doesn’t help” and refuses to take it. What should 16.Richard with agoraphobia has been symptom-free for 4 months.
the nurse say or do? Classic signs and symptoms of phobias include:
a. Withhold the drug. b. Record the client’s response. c. Encourage a. Insomnia and an inability to concentrate. b. Severe anxiety and
the client to tell the doctor. d. Suggest that it takes awhile before fear. c. Depression and weight loss. d. Withdrawal and failure to
seeing the results. distinguish reality from fantasy.
7. Dervid, an adolescent has a history of truancy from school, 17.Which medications have been found to help reduce or eliminate
running away from home and “barrowing” other people’s things panic attacks?
without their permission. The adolescent denies stealing, a. Antidepressants b. Anticholinergics c. Antipsychotics d. Mood
rationalizing instead that as long as no one was using the items, it stabilizers
was all right to borrow them. It is important for the nurse to 18.A client seeks care because she feels depressed and has gained
understand the psychodynamically, this behavior may be largely weight. To treat her atypical depression, the physician prescribes
attributed to a developmental defect related to the: tranylcypromine sulfate (Parnate), 10 mg by mouth twice per day.
a. Id b. Ego c. Superego d. Oedipal complex
When this drug is used to treat atypical depression, what is its onset a. “It is the voice of your conscience, which only you can control.” b.
of action? “No, I do not hear your voices, but I believe you can hear them”. c.
a. 1 to 2 days b. 3 to 5 days c. 6 to 8 days “The voices are coming from within you and only you can hear
Nursing Crib – Student Nurses’ Community 96 them.” d. “Oh, the voices are a symptom of your illness; don’t pay
d. 10 to 14 days any attention to them.”
19.A 65 years old client is in the first stage of Alzheimer's disease. 32.The nurse is aware that the side effect of electroconvulsive
Nurse Patricia should plan to focus this client's care on: therapy that a client may experience:
a. Offering nourishing finger foods to help maintain the client's a. Loss of appetite b. Postural hypotension c. Confusion for a time
nutritional status. b. Providing emotional support and individual after treatment d. Complete loss of memory for a time
counseling. c. Monitoring the client to prevent minor illnesses from 33.A dying male client gradually moves toward resolution of feelings
turning into major problems. d. Suggesting new activities for the regarding impending death. Basing care on the theory of Kubler-
client and family to do together. Ross, Nurse Trish plans to use nonverbal interventions when
20.The nurse is assessing a client who has just been admitted to the assessment reveals that the client is in the:
emergency department. Which signs would suggest an overdose of a. Anger stage b. Denial stage c. Bargaining stage
an antianxiety agent? Nursing Crib – Student Nurses’ Community 99
a. Combativeness, sweating, and confusion b. Agitation, d. Acceptance stage
hyperactivity, and grandiose ideation c. Emotional lability, euphoria, 34.The outcome that is unrelated to a crisis state is:
and impaired memory d. Suspiciousness, dilated pupils, and a. Learning more constructive coping skills b. Decompensation to a
increased blood pressure lower level of functioning. c. Adaptation and a return to a prior level
21.The nurse is caring for a client diagnosed with antisocial of functioning. d. A higher level of anxiety continuing for more than
personality disorder. The client has a history of fighting, cruelty to 3 months.
animals, and stealing. Which of the following traits would the nurse 35.Miranda a psychiatric client is to be discharged with orders for
be most likely to uncover during assessment? haloperidol (haldol) therapy. When developing a teaching plan for
a. History of gainful employment b. Frequent expression of guilt discharge, the nurse should include cautioning the client against:
regarding antisocial behavior c. Demonstrated ability to maintain a. Driving at night b. Staying in the sun c. Ingesting wines and
close, stable relationships d. A low tolerance for frustration cheeses d. Taking medications containing aspirin
22.Nurse Amy is providing care for a male client undergoing opiate 36.Jen a nursing student is anxious about the upcoming board
withdrawal. Opiate withdrawal causes severe physical discomfort examination but is able to study intently and does not become
and can be life-threatening. To minimize these effects, opiate users distracted by a roommate’s talking and loud music. The student’s
are commonly detoxified with: ability to ignore distractions and to focus on studying demonstrates:
a. Barbiturates b. Amphetamines c. Methadone d. Benzodiazepines a. Mild-level anxiety b. Panic-level anxiety c. Severe-level anxiety d.
23.Nurse Cristina is caring for a client who experiences false sensory Moderate-level anxiety
perceptions with no basis in reality. These perceptions are known as: 37.When assessing a premorbid personality characteristics of a client
a. Delusions b. Hallucinations with a major depression, it would be unusual for the nurse to find
Nursing Crib – Student Nurses’ Community 97 that this client demonstrated:
c. Loose associations d. Neologisms a. Rigidity b. Stubbornness c. Diverse interest d. Over meticulousness
24.Nurse Marco is developing a plan of care for a client with 38.Nurse Krina recognizes that the suicidal risk for depressed client
anorexia nervosa. Which action should the nurse include in the plan? is greatest:
a. Restricts visits with the family and friends until the client begins to a. As their depression begins to improve b. When their depression is
eat. b. Provide privacy during meals. c. Set up a strict eating plan for most severe c. Before nay type of treatment is started d. As they
the client. d. Encourage the client to exercise, which will reduce her lose interest in the environment
anxiety. Nursing Crib – Student Nurses’ Community 100
25.Tim is admitted with a diagnosis of delusions of grandeur. The 39.Nurse Kate would expect that a client with vascular dementis
nurse is aware that this diagnosis reflects a belief that one is: would experience:
a. Highly important or famous. b. Being persecuted c. Connected to a. Loss of remote memory related to anoxia b. Loss of abstract
events unrelated to oneself d. Responsible for the evil in the world. thinking related to emotional state c. Inability to concentrate related
26.Nurse Jen is caring for a male client with manic depression. The to decreased stimuli d. Disturbance in recalling recent events related
plan of care for a client in a manic state would include: to cerebral hypoxia.
a. Offering a high-calorie meals and strongly encouraging the client 40.Josefina is to be discharged on a regimen of lithium carbonate. In
to finish all food. b. Insisting that the client remain active through the teaching plan for discharge the nurse should include:
the day so that he’ll sleep at night. c. Allowing the client to exhibit a. Advising the client to watch the diet carefully b. Suggesting that
hyperactive, demanding, manipulative behavior without setting the client take the pills with milk c. Reminding the client that a CBC
limits. d. Listening attentively with a neutral attitude and avoiding must be done once a month. d. Encouraging the client to have blood
power struggles. levels checked as ordered.
27.Ramon is admitted for detoxification after a cocaine overdose. 41.The psychiatrist orders lithium carbonate 600 mg p.o t.i.d for a
The client tells the nurse that he frequently uses cocaine but that he female client. Nurse Katrina would be aware that the teaching about
can control his use if he chooses. Which coping mechanism is he the side effects of this drug were understood when the client state,
using? “I will call my doctor immediately if I notice any:
a. Withdrawal b. Logical thinking c. Repression d. Denial a. Sensitivity to bright light or sun b. Fine hand tremors or slurred
28.Richard is admitted with a diagnosis of schizotypal personality speech c. Sexual dysfunction or breast enlargement d. Inability to
disorder. Which signs would this client exhibit during social urinate or difficulty when urinating
situations? 42.Nurse Mylene recognizes that the most important factor
a. Aggressive behavior b. Paranoid thoughts necessary for the establishment of trust in a critical care area is:
Nursing Crib – Student Nurses’ Community 98 a. Privacy b. Respect c. Empathy d. Presence
c. Emotional affect d. Independence needs 43.When establishing an initial nurse-client relationship, Nurse Hazel
29.Nurse Mickey is caring for a client diagnosed with bulimia. The should explore with the client the:
most appropriate initial goal for a client diagnosed with bulimia is to: a. Client’s perception of the presenting problem. b. Occurrence of
a. Avoid shopping for large amounts of food. b. Control eating fantasies the client may experience. c. Details of any ritualistic acts
impulses. c. Identify anxiety-causing situations d. Eat only three carried out by the client d. Client’s feelings when external; controls
meals per day. are instituted.
30.Rudolf is admitted for an overdose of amphetamines. When 44.Tranylcypromine sulfate (Parnate) is prescribed for a depressed
assessing the client, the nurse should expect to see: client who has not responded to the tricyclic antidepressants. After
a. Tension and irritability b. Slow pulse c. Hypotension d. teaching the client about the medication, Nurse Marian evaluates
Constipation that learning has occurred when the client states, “I will avoid:
31.Nicolas is experiencing hallucinations tells the nurse, “The voices Nursing Crib – Student Nurses’ Community 101
are telling me I’m no good.” The client asks if the nurse hears the a. Citrus fruit, tuna, and yellow vegetables.” b. Chocolate milk, aged
voices. The most appropriate response by the nurse would be: cheese, and yogurt’” c. Green leafy vegetables, chicken, and milk.” d.
Whole grains, red meats, and carbonated soda.”
45.Nurse John is a aware that most crisis situations should resolve in a. Occasional irritable outbursts. b. Impaired communication. c. Lack
about: of spontaneity. d. Inability to perform self-care activities.
a. 1 to 2 weeks b. 4 to 6 weeks c. 4 to 6 months d. 6 to 12 months 58.Isabel with a diagnosis of depression is started on imipramine
46. Nurse Judy knows that statistics show that in adolescent suicide (Tofranil), 75 mg by mouth at bedtime. The nurse should tell the
behavior: client that:
a. Females use more dramatic methods than males b. Males account a. This medication may be habit forming and will be discontinued as
for more attempts than do females c. Females talk more about soon as the client feels better. b. This medication has no serious
suicide before attempting it d. Males are more likely to use lethal adverse effects.
methods than are females Nursing Crib – Student Nurses’ Community 104
47.Dervid with paranoid schizophrenia repeatedly uses profanity c. The client should avoid eating such foods as aged cheeses, yogurt,
during an activity therapy session. Which response by the nurse and chicken livers while taking the medication. d. This medication
would be most appropriate? may initially cause tiredness, which should become less bothersome
a. "Your behavior won't be tolerated. Go to your room over time.
immediately." b. "You're just doing this to get back at me for making 59.Kathleen is admitted to the psychiatric clinic for treatment of
you come to therapy." c. "Your cursing is interrupting the activity. anorexia nervosa. To promote the client's physical health, the nurse
Take time out in your room for 10 minutes." d. "I'm disappointed in should plan to: a. Severely restrict the client's physical activities. b.
you. You can't control yourself even for a few minutes." Weigh the client daily, after the evening meal. c. Monitor vital signs,
48.Nurse Maureen knows that the nonantipsychotic medication serum electrolyte levels, and acid-base balance. d. Instruct the client
used to treat some clients with schizoaffective disorder is: to keep an accurate record of food and fluid intake.
a. phenelzine (Nardil) b. chlordiazepoxide (Librium) c. lithium 60.Celia with a history of polysubstance abuse is admitted to the
carbonate (Lithane) d. imipramine (Tofranil) facility. She complains of nausea and vomiting 24 hours after
49.Which information is most important for the nurse Trinity to admission. The nurse assesses the client and notes piloerection,
include in a teaching plan for a male schizophrenic client taking pupillary dilation, and lacrimation. The nurse suspects that the client
clozapine (Clozaril)? is going through which of the following withdrawals?
a. Monthly blood tests will be necessary. b. Report a sore throat or a. Alcohol withdrawal b. Cannibis withdrawal c. Cocaine withdrawal
fever to the physician immediately. d. Opioid withdrawal
Nursing Crib – Student Nurses’ Community 102 61.Mr. Garcia, an attorney who throws books and furniture around
c. Blood pressure must be monitored for hypertension. d. Stop the the office after losing a case is referred to the psychiatric nurse in
medication when symptoms subside. the law firm's employee assistance program. Nurse Beatriz knows
50.Ricky with chronic schizophrenia takes neuroleptic medication is that the client's behavior most likely represents the use of which
admitted to the psychiatric unit. Nursing assessment reveals rigidity, defense mechanism?
fever, hypertension, and diaphoresis. These findings suggest which a. Regression b. Projection c. Reaction-formation d.
lifethreatening reaction: Intellectualization
a. Tardive dyskinesia. b. Dystonia. c. Neuroleptic malignant 62.Nurse Anne is caring for a client who has been treated long term
syndrome. d. Akathisia. with antipsychotic medication. During the assessment, Nurse Anne
51.Which nursing intervention would be most appropriate if a male checks the client for tardive dyskinesia. If tardive dyskinesia is
client develop orthostatic hypotension while taking amitriptyline present, Nurse Anne would most likely observe:
(Elavil)? a. Abnormal movements and involuntary movements of the mouth,
a. Consulting with the physician about substituting a different type tongue, and face. b. Abnormal breathing through the nostrils
of antidepressant. b. Advising the client to sit up for 1 minute before accompanied by a “thrill.” c. Severe headache, flushing, tremors, and
getting out of bed. c. Instructing the client to double the dosage until ataxia. d. Severe hypertension, migraine headache,
the problem resolves. d. Informing the client that this adverse Nursing Crib – Student Nurses’ Community 105
reaction should disappear within 1 week. 63.Dennis has a lithium level of 2.4 mEq/L. The nurse immediately
52.Mr. Cruz visits the physician's office to seek treatment for would assess the client for which of the following signs or
depression, feelings of hopelessness, poor appetite, insomnia, symptoms?
fatigue, low selfesteem, poor concentration, and difficulty making a. Weakness b. Diarrhea c. Blurred vision d. Fecal incontinence
decisions. The client states that these symptoms began at least 2 64.Nurse Jannah is monitoring a male client who has been placed
years ago. Based on this report, the nurse Tyfany suspects: inrestraints because of violent behavior. Nurse determines that it
a. Cyclothymic disorder. b. Atypical affective disorder. c. Major will be safe to remove the restraints when:
depression. d. Dysthymic disorder. a. The client verbalizes the reasons for the violent behavior. b. The
53.After taking an overdose of phenobarbital (Barbita), Mario is client apologizes and tells the nurse that it will never happen again.
admitted to the emergency department. Dr. Trinidad prescribes c. No acts of aggression have been observed within 1 hour after the
activated charcoal (Charcocaps) to be administered by mouth release of two of the extremity restraints. d. The administered
immediately. Before administering the dose, the nurse verifies the medication has taken effect.
dosage ordered. What is the usual minimum dose of activated 65.Nurse Irish is aware that Ritalin is the drug of choice for a child
charcoal? with ADHD. The side effects of the following may be noted by the
a. 5 g mixed in 250 ml of water b. 15 g mixed in 500 ml of water c. 30 nurse:
g mixed in 250 ml of water d. 60 g mixed in 500 ml of water a. Increased attention span and concentration b. Increase in appetite
Nursing Crib – Student Nurses’ Community 103 c. Sleepiness and lethargy d. Bradycardia and diarrhea
54.What herbal medication for depression, widely used in Europe, is 66.Kitty, a 9 year old child has very limited vocabulary and
now being prescribed in the United States? interaction skills. She has an I.Q. of 45. She is diagnosed to have
a. Ginkgo biloba b. Echinacea c. St. John's wort d. Ephedra Mental retardation of this classification:
55.Cely with manic episodes is taking lithium. Which electrolyte level a. Profound b. Mild c. Moderate d. Severe
should the nurse check before administering this medication? 67.The therapeutic approach in the care of Armand an autistic child
a. Calcium b. Sodium c. Chloride d. Potassium include the following EXCEPT:
56.Nurse Josefina is caring for a client who has been diagnosed with a. Engage in diversionary activities when acting -out b. Provide an
delirium. Which statement about delirium is true? atmosphere of acceptance c. Provide safety measures d. Rearrange
a. It's characterized by an acute onset and lasts about 1 month. b. the environment to activate the child
It's characterized by a slowly evolving onset and lasts about 1 week. 68.Jeremy is brought to the emergency room by friends who state
c. It's characterized by a slowly evolving onset and lasts about 1 that he took something an hour ago. He is actively hallucinating,
month. d. It's characterized by an acute onset and lasts hours to a agitated, with
number of days. Nursing Crib – Student Nurses’ Community 106
57.Edward, a 66 year old client with slight memory impairment and irritated nasal septum.
poor concentration is diagnosed with primary degenerative a. Heroin b. Cocaine c. LSD d. Marijuana
dementia of the Alzheimer's type. Early signs of this dementia 69.Nurse Pauline is aware that Dementia unlike delirium is
include subtle personality changes and withdrawal from social characterized by:
interactions. To assess for progression to the middle stage of a. Slurred speech b. Insidious onset c. Clouding of consciousness d.
Alzheimer's disease, the nurse should observe the client for: Sensory perceptual change
70.A 35 year old female has intense fear of riding an elevator. She d. I’ve lost my phobia for water”
claims “ As if I will die inside.” The client is suffering from: a. 84. Mark, with a diagnosis of generalized anxiety disorder wants to
Agoraphobia b. Social phobia c. Claustrophobia d. Xenophobia stop taking his lorazepam (Ativan). Which of the following important
71.Nurse Myrna develops a counter-transference reaction. This is facts should nurse Betty discuss with the client about discontinuing
evidenced by: the medication?
a. Revealing personal information to the client b. Focusing on the a. Stopping the drug may cause depression b. Stopping the drug
feelings of the client. c. Confronting the client about discrepancies in increases cognitive abilities c. Stopping the drug decreases sleeping
verbal or non-verbal behavior d. The client feels angry towards the difficulties d. Stopping the drug can cause withdrawal symptoms
nurse who resembles his mother. 85. Jennifer, an adolescent who is depressed and reported by his
72.Tristan is on Lithium has suffered from diarrhea and vomiting. parents as having difficulty in school is brought to the community
What should the nurse in-charge do first: mental health center to be evaluated. Which of the following other
a. Recognize this as a drug interaction b. Give the client Cogentin c. health problems would the nurse suspect?
Reassure the client that these are common side effects of lithium a. Anxiety disorder b. Behavioral difficulties c. Cognitive impairment
therapy d. Hold the next dose and obtain an order for a stat serum d. Labile moods
lithium level 86. Ricardo, an outpatient in psychiatric facility is diagnosed with
73.Nurse Sarah ensures a therapeutic environment for all the client. dysthymic disorder. Which of the following statement about
Which of the following best describes a therapeutic milieu? dysthymic disorder is true?
a. A therapy that rewards adaptive behavior b. A cognitive approach a. It involves a mood range from moderate depression to hypomania
to change behavior c. A living, learning or working environment. b. It involves a single manic depression c. It’s a form of depression
Nursing Crib – Student Nurses’ Community 107 that occurs in the fall and winter d. It’s a mood disorder similar to
d. A permissive and congenial environment major depression but of mild to moderate severity
74.Anthony is very hostile toward one of the staff for no apparent
reason. He is manifesting: 87. The nurse is aware that the following ways in vascular dementia
a. Splitting b. Transference c. Countertransference d. Resistance different from Alzheimer’s disease is:
75.Marielle, 17 years old was sexually attacked while on her way a. Vascular dementia has more abrupt onset b. The duration of
home from school. She is brought to the hospital by her mother. vascular dementia is usually brief c. Personality change is common
Rape is an example of which type of crisis: in vascular dementia d. The inability to perform motor activities
a. Situational b. Adventitious c. Developmental d. Internal occurs in vascular dementia
76.Nurse Greta is aware that the following is classified as an Axis I 88. Loretta, a newly admitted client was diagnosed with delirium
disorder by the Diagnosis and Statistical Manual of Mental and has history of hypertension and anxiety. She had been taking
Disorders, Text Revision (DSM-IV-TR) is: digoxin, furosemide (Lasix), and diazepam (Valium) for anxiety. This
a. Obesity b. Borderline personality disorder c. Major depression d. client’s impairment may be related to which of the following
Hypertension conditions?
77.Katrina, a newly admitted is extremely hostile toward a staff a. Infection b. Metabolic acidosis
member she has just met, without apparent reason. According to Nursing Crib – Student Nurses’ Community 110
Freudian theory, the nurse should suspect that the client is c. Drug intoxication d. Hepatic encephalopathy
experiencing which of the following phenomena? 89. Nurse Ron enters a client’s room, the client says, “They’re
a. Intellectualization b. Transference c. Triangulation d. Splitting crawling on my sheets! Get them off my bed!” Which of the
78.An 83year-old male client is in extended care facility is anxious following assessment is the most accurate?
most of the time and frequently complains of a number of vague a. The client is experiencing aphasia b. The client is experiencing
symptoms that interfere with his ability to eat. These symptoms dysarthria c. The client is experiencing a flight of ideas d. The client is
indicate which of the following disorders? experiencing visual hallucination
a. Conversion disorder b. Hypochondriasis c. Severe anxiety 90. Which of the following descriptions of a client’s experience and
Nursing Crib – Student Nurses’ Community 108 behavior can be assessed as an illusion?
d. Sublimation a. The client tries to hit the nurse when vital signs must be taken b.
79. Charina, a college student who frequently visited the health The client says, “I keep hearing a voice telling me to run away” c. The
center during the past year with multiple vague complaints of GI client becomes anxious whenever the nurse leaves the bedside d.
symptoms before course examinations. Although physical causes The client looks at the shadow on a wall and tells the nurse she sees
have been eliminated, the student continues to express her belief frightening faces on the wall.
that she has a serious illness. These symptoms are typically of which 91. During conversation of Nurse John with a client, he observes that
of the following disorders? the client shift from one topic to the next on a regular basis. Which
a. Conversion disorder b. Depersonalization c. Hypochondriasis d. of the following terms describes this disorder?
Somatization disorder a. Flight of ideas b. Concrete thinking c. Ideas of reference d. Loose
80. Nurse Daisy is aware that the following pharmacologic agents association
are sedativehypnotic medication is used to induce sleep for a client 92. Francis tells the nurse that her coworkers are sabotaging the
experiencing a sleep disorder is: computer. When the nurse asks questions, the client becomes
a. Triazolam (Halcion) b. Paroxetine (Paxil)\ c. Fluoxetine (Prozac) d. argumentative. This behavior shows personality traits associated
Risperidone (Risperdal) with which of the following personality disorder?
81. Aldo, with a somatoform pain disorder may obtain secondary a. Antisocial b. Histrionic c. Paranoid d. Schizotypal
gain. Which of the following statement refers to a secondary gain? 93. Which of the following interventions is important for a Cely
a. It brings some stability to the family b. It decreases the experiencing with paranoid personality disorder taking olanzapine
preoccupation with the physical illness c. It enables the client to (Zyprexa)?
avoid some unpleasant activity d. It promotes emotional support or a. Explain effects of serotonin syndrome b. Teach the client to watch
attention for the client for extrapyramidal adverse reaction
82. Dervid is diagnosed with panic disorder with agoraphobia is Nursing Crib – Student Nurses’ Community 111
talking with the nurse in-charge about the progress made in c. Explain that the drug is less affective if the client smokes d. Discuss
treatment. Which of the following statements indicates a positive the need to report paradoxical effects such as euphoria
client response? 94. Nurse Alexandra notices other clients on the unit avoiding a
a. “I went to the mall with my friends last Saturday” b. “I’m client diagnosed with antisocial personality disorder. When
hyperventilating only when I have a panic attack” c. “Today I decided discussing appropriate behavior in group therapy, which of the
that I can stop taking my medication” d. “Last night I decided to eat following comments is expected about this client by his peers?
more than a bowl of cereal” a. Lack of honesty b. Belief in superstition c. Show of temper
83. The effectiveness of monoamine oxidase (MAO) inhibitor drug tantrums d. Constant need for attention
therapy in a client with posttraumatic stress disorder can be 95. Tommy, with dependent personality disorder is working to
demonstrated by which of the following client self –reports? increase his selfesteem. Which of the following statements by the
a. “I’m sleeping better and don’t have nightmares” b. “I’m not losing Tommy shows teaching was successful?
my temper as much” c. “I’ve lost my craving for alcohol” a. “I’m not going to look just at the negative things about myself” b.
Nursing Crib – Student Nurses’ Community 109 “I’m most concerned about my level of competence and progress” c.
“I’m not as envious of the things other people have as I used to be”
d. “I find I can’t stop myself from taking over things other should be
doing”
96. Norma, a 42-year-old client with a diagnosis of chronic
undifferentiated schizophrenia lives in a rooming house that has a
weekly nursing clinic. She scratches while she tells the nurse she
feels creatures eating away at her skin. Which of the following
interventions should be done first?
a. Talk about his hallucinations and fears b. Refer him for
anticholinergic adverse reactions c. Assess for possible physical
problems such as rash d. Call his physician to get his medication
increased to control his psychosis
97. Ivy, who is on the psychiatric unit is copying and imitating the
movements of her primary nurse. During recovery, she says, “I
thought the nurse was my mirror. I felt connected only when I saw
my nurse.” This behavior is known by which of the following terms?
a. Modeling b. Echopraxia c. Ego-syntonicity d. Ritualism
Nursing Crib – Student Nurses’ Community 112
98. Jun approaches the nurse and tells that he hears a voice telling
him that he’s evil and deserves to die. Which of the following terms
describes the client’s perception?
a. Delusion b. Disorganized speech c. Hallucination d. Idea of
reference
99. Mike is admitted to a psychiatric unit with a diagnosis of
undifferentiated schizophrenia. Which of the following defense
mechanisms is probably used by mike?
a. Projection b. Rationalization c. Regression d. Repression
100. Rocky has started taking haloperidol (Haldol). Which of the
following instructions is most appropriate for Ricky before taking
haloperidol?
a. Should report feelings of restlessness or agitation at once b. Use a
sunscreen outdoors on a year-round basis c. Be aware you’ll feel
increased energy taking this drug d. This drug will indirectly control
essential hypertension

Potrebbero piacerti anche